Vous êtes sur la page 1sur 31

Obstetrics,

Gynaecology
and Reproductive
Medicine
Obstetrics, Gynaecology and Reproductive Medicine is a great revision guide for the MRCOG and beyond.
Obstetrics, Gynaecology and Reproductive Medicine is an authoritative and comprehensive resource that provides all obstetricians,
gynaecologists and specialists in reproductive medicine with ready access to up-to-date information on all aspects of obstetrics
and gynaecology. Over a three-year cycle of 36 issues, the emphasis of the journal is on the clear and concise presentation of
information of direct clinical relevance to specialists in the field and candidates studying for MRCOG Part II. Each volume contains
review articles on obstetric and gynaecological topics. The journal is invaluable for specialists in obstetrics and gynaecology,
both in their role as trainers of MRCOG candidates and in keeping up to date across the broad span of the subject area. Over
any three-year period, a subscription will ensure access to up-to-date, understandable information on the full range of obstetrics,
gynaecology and reproductive medicine topics.

Editor-in-Chief
Alec McEwan BA BM BCh MD MRCOG
Consultant in Fetal and Maternal Medicine, Department of Obstetrics and Gynaecology,
Queens Medical Centre, Nottingham, UK

Associate Editors
Sabaratnam Arulkumaran MBBS MD PhD FRCS (Ed) FRCOG
Professor of Obstetrics and Gynaecology,
Department of Obstetrics and Gynaecology,
St. Georges Hospital Medical School, London, UK

Tahir A Mahmood MD FRCOG FRCPI MBA FACOG(Hon)


Consultant Obstetrician and Gynaecologist,
Victoria Hospital, Kirkcaldy, Fife, UK

Philip N Baker FRCOG, FMedSci


Pro-Vice-Chancellor and Head of the College of Medicine,
Biological Sciences and Psychology,
Dean of the School of Medicine, University of Leicester, UK

Fiona Reid MD MRCOG


Consultant Urogynaecologist,
St Marys Hospital, Manchester, UK

Shreelata Datta BSc(Hons) MBBS MRCOG LLM


Consultant Obstetrician and Gynaecologist,
Kings College Hospital, London, UK

Mahmood I Shafi MB BCh MD DA FRCOG


Consultant Gynaecological Surgeon and Oncologist,
Addenbrookes Hospital, Cambridge, UK

Trainee Editor
Catherine Aiken MB/BChir MA PhD MRCP MRCOG
Specialist Registrar (ST5) and Academic Clinical Lecturer in Obstetrics and Gynaecology,
Addenbrookes Hospital, Cambridge, UK
Obstetrics, Gynaecology and Reproductive Medicine has an eminent editorial board, all of whom are recognized experts in their field.
Visit our website at: www.obstetrics-gynaecology-journal.com for previous issues, subscription information and further details.

REVIEW

Vaginal discharge

Causes of vaginal discharge include: physiological discharge,


bacterial vaginosis (BV), vulvovaginal candidiasis (VVC), T.
vaginalis (TV), chlamydia, gonorrhoea, herpes simplex virus,
foreign body (e.g. retained tampon and condom), irritants (e.g.
perfumes or deodorants), atrophic vaginitis, fistulae and tumours
affecting the vulva, vagina and cervix.

Alexandra Rice
Mohamed ElWerdany
Essam Hadoura
Tahir Mahmood

History and examination


Vaginal discharge is a clinical feature not a diagnosis. History
and examination of the patient should be the first line in deciding
whether investigations and treatment are required. A routine
gynaecological history should be obtained including parity,
smear history, sexual history and current contraception. Sexual
history should dictate the need for discussion regarding full STI
screening. With regards to the vaginal discharge, its onset,
duration, timing related to menstrual cycle, odour, colour, consistency and any exacerbating factors should be noted. Associated symptoms including itch, discomfort, pain, dysuria,
dyspareunia and irregular bleeding should be enquired.
A gynaecological examination consists of inspection,
bimanual examination and obtaining appropriate vaginal swabs.
The inspection part, includes both, a general external inspection
of the vulva and the perineal region, and an internal inspection of
the vagina and cervix with the aid of a speculum. Bimanual examination will give the gynaecologist an idea about the position,
size and mobility of the uterus as well as the presence of any
adnexal masses. Vaginal swab will help in the diagnosis of
pathogens that may be responsible for the abnormal discharge.

Abstract
Vaginal discharge is a common presenting symptom at gynaecology
and sexual health clinics and in general practice. It is usually physiological and is subject to hormonal variations in consistency and quantity. With this in mind, appropriate diagnosis and treatment of
abnormal vaginal discharge can be challenging. Concurrent pregnancy
can also complicate the situation. Some pathological conditions may
contribute to vaginal discharge, including cervicitis, aerobic vaginitis,
atrophic vaginitis and mucoid ectopy. We mainly focus on the three
most prevalent pathological causes namely; bacterial vaginosis, vulvovaginal candidiasis and Trichomonas vaginalis and will also provide a
brief overview of atypical inammatory vaginitis as well. Obtaining a
methodical and detailed history from the patient should give the majority of the information required. Examination and analysis of discharge
with swabs are a useful adjunct to aid diagnosis. Once a diagnosis is
made, appropriate treatment must then be instigated and in some
cases partner notication and treatment may also be required.

Keywords bacterial vaginosis; candidiasis; desquamated inammatory vaginitis (DIV); HIV; pregnancy; trichomonas; vaginal discharge

Who to swab?
Introduction

Some patients can be given treatment without the need for full
investigations. A patient who complains with a first episode of
vaginal discharge with a clear clinical evidence of either vulvovaginal candidiasis (VVC) or bacterial vaginosis (BV), and no
other risk factors, can be given empirical treatment without
further investigations. However, the following risk factors
require further investigations:
 High STI risk (past history of STI, multiple sexual partners,
sharing needles and intravenous drug use)
 Symptoms suggestive of an alternative cause (e.g. vaginal
bleeding and urinary or bowel symptoms)
 Recent gynaecological procedure
 BV associated with pregnancy
Indications to obtain a swab:
 STI risk/requesting STI screening
 Symptoms suggestive of upper genital tract infection
 Postpartum, post-miscarriage, TOP or recent instrumentation of the uterus
 Recurrent symptoms despite treatment
 Abnormal symptoms of unknown cause
 Cervicitis found on examination.

Vaginal discharge is a common condition. Discharge is mainly


physiological. It is affected by hormonal variation throughout the
lifetime of females. Diagnosis of abnormal discharge is quiet
challenging. Conditions such as pregnancy and co-morbidities
like diabetes mellitus make the management really challenging.
Pathologies such as vaginitis, cervicitis or cervical atopy may
also contribute to vaginal discharge. The most common causes of
vaginal discharge are bacterial vaginosis, candidiasis and Trichomonas vaginalis.

Alexandra Rice MRCOG is a Specialty Trainee in Obstetrics and


Gynaecology, South East of Scotland, Victoria Hospital, Kirkcaldy,
Fife, UK. Conict of interest: none.
Mohamed ElWerdany MBBCh MSc is a Specialty Trainee in Obstetrics
and Gynaecology, South East of Scotland, Victoria Hospital,
Kirkcaldy, Fife, UK. Conict of interest: none.
Essam Hadoura FRCOG is a Consultant Obstetrician and
Gynaecologist at Victoria Hospital, Kirkcaldy, Fife, and Honorary
Senior Lecturer, University of St. Andrews, St Andrews, UK. Conict
of interest: none.

Taking appropriate swabs


There should be appropriate documentation sent with the swab
to aid proper testing in the lab. The site that was sampled should
be well documented. Symptoms, any recent treatments (e.g.
systemic antibiotics), pregnancy status and any recent

Tahir Mahmood MD FRCPI FACOG FRCPE FRCOG is a Consultant


Obstetrician and Gynaecologist at Victoria Hospital, Kirkcaldy, Fife,
UK. Conict of interest: none.

OBSTETRICS, GYNAECOLOGY AND REPRODUCTIVE MEDICINE 26:11

317

2016 Published by Elsevier Ltd.

REVIEW

amplification testing) took its place. Culture should be


undertaken in patients with negative microscopy despite
symptoms and those with apparent recurrent disease.

gynaecological procedures should be mentioned. Different labs


will have different capabilities with regards to the specific testing
they offer, therefore you should check your local laboratory prior
to sending samples.

G. NAAT (nucleic acid amplication tests)


Is effective in detecting Chlamydia and gonorrhoea. Reported
detection rates of 100% with BV although its use in the diagnosis
of BV is not widespread.

A. High vaginal swab (HVS)


A swab taken from the lateral wall or posterior fornix of the
vagina under direct vision. The swab should then be placed in
Amies transport medium with charcoal. The high number of
commensal bacteria that reside in the vagina can often make
interpretation of results difficult and these should be interpreted
in the context of the clinical picture. It should only be obtained if
there is a clinical need for testing and not as a routine with
speculum examination. It should be obtained when symptoms do
not lead to a diagnosis. Abnormal discharge in pregnancy, postpartum, post-termination and post instrumentation should always be swabbed. Similarly, if there is recurrence of symptoms
or possible treatment failure then a swab should be obtained.

H. Intrauterine contraceptive devices (IUCDs)


The entire device should be sent to microbiology. The presence
of an IUCD may be associated with PID. Infections may be polymicrobial with the isolation of both Gram positive and Gram
negative aerobic and anaerobic organisms. Actinomyces species,
particularly Actinomyces israelii, may be significant isolates. We
recommend that IUCDs are only cultured where there are clinical
indications of PID or other inflammatory conditions.

Physiological discharge
B. Endocervical swab (ECS)
The cervical os should be cleaned with a disposable swab and
discarded. The ECS should then be inserted into the cervical os
and rotated firmly. The swab should then be placed in Amies
transport medium with charcoal. It is mainly used in the investigation of Chlamydia and gonorrhoea. The swab is sent for
NAAT (nucleic acid amplification testing). Some labs are now
analysing these samples for BV and TV.

Physiological vaginal discharge is a normal finding in all women


and most commonly described as an inoffensive discharge. The
fluctuating levels of oestrogen and progesterone during the
menstrual cycle greatly affects the consistency and composition
of the physiological discharge. Oestrogen makes the discharge
thin and clear for easy passage of sperm through the cervix at the
time of ovulation. Progesterone makes the discharge thick and
sticky after ovulation.
The vaginal environment maintains its stability by the action
of commensal organisms. Lactobacilli colonize the vagina since
puberty under the influence of oestrogen. These are responsible
for converting glycogen to lactic acid, maintaining a vaginal pH
of around 4.5. Other commensal organisms are streptococci,
enterococci and coagulase negative staphylococci. A few other
organisms which are part of the normal flora, but are associated
with vaginal infections, include, anaerobic Bacteroides, anaerobic cocci, Gardenella vaginalis, Candida, Ureaplasma urealyticum and Mycoplasma species.

C. pH testing
A swab from the lateral vaginal wall is placed on a narrow
spectrum litmus paper. BV and TV (T. vaginalis) will have pH
>4.5. There is good evidence that clinical features and measurement of vaginal pH is a sensitive (but not specific) predictor.
If HVS and ECS are also obtained, there is an increased accuracy
of diagnosis. Therefore, a swab should be obtained if features are
not suggestive of BV/VVC.
D. Microscopy
Wet microscopy requires a certain level of expertize and technical skills for proper implementation. A small sample of the
discharge should be placed on two ends of a slide. Normal saline
is put on one end and potassium hydroxide on the other. A cover
slip is placed on the slide and these are visualized under a microscope. This test is of varying sensitivities depending on the
offending organism:
 70% sensitivity for TV (TV swabs need to be processed
with wet microscopy within 6 hours).
 Saline microscopy will show spores/pseudohyphae in 40
e60% of Candida
 30e50% sensitivity for Gonorrhoea

Bacterial vaginosis
Aetiology and transmission
Bacterial vaginosis is the commonest cause of vaginal discharge
in women; during their childbearing period. It has been found in
postmenopausal women and rarely prepubertal. It is more
common in black African and American women (45e55%).
Caucasian women have a prevalence of approximately 5e15%.
Previously, it was considered as a harmless finding, but we now
know it to be associated with many pathological conditions such
as puerperal endometritis, preterm labour, premature rupture of
membranes, PID and UTI.
The condition is associated with a prevalence of the anaerobic
species in preference to the normal Lactobacillus species. Organisms associated with bacterial vaginosis include the Prevotella
species, G. vaginalis, Mobiluncus species, Peptostreptococcus
species and Mycoplasma hominis. Some women experience the
change in the microorganism environment really abruptly, while
others take longer time interval to feel the change.
Bacterial vaginosis is experienced more in patients with
multiple sexual partners, no use of condoms or douching. Most

E. Gram stain
 Commonly used for the diagnosis and grading of BV
 65e68% detection in symptomatic Candida
F. Culture
 Candida grows best on Sabouraud agar (95% growth on
culture). Its growth can be classified as light, medium or
heavy. Culture was considered the gold standard for
detecting T. Vaginalis before NAAT (nucleic acid

OBSTETRICS, GYNAECOLOGY AND REPRODUCTIVE MEDICINE 26:11

318

2016 Published by Elsevier Ltd.

REVIEW

 Preoperative for vaginal surgery


 Pregnant women, if further investigated with direct microscopy (due to persistent negative gram stain findings
and still symptomatic or failure of treatment) and found
positive.
 Treatment individualized in patients with positive direct
microscopy without symptom.
Recommended regimens:
 500 mg Metronidazole: oral tablets, twice daily for 7 days,
or
 2 gm Metronidazole: Single oral dose, or
 0.75% Metronidazole gel: one full applicator (5 g) intravaginally, once daily for 5 days, or
 2% Clindamycin cream: one full applicator (5 g) intravaginally at bedtime for 7 days
Alcohol, sexual intercourse or vaginal douching may hinder
the effectiveness of treatment. Alcohol should not be consumed
during treatment up to 24 hours after completion of metronidazole course, as it may precipitate a disulfiram-like reaction.
Clindamycin cream may affect latex condoms and diaphragms
and reduce their effectiveness for up to 5 days after its use.
Vaginal douching increases the risk of recurrence and insufficient
data supports their use for treatment or symptom relief.
Some studies have further evaluated the effectiveness of
intravaginal lactobacillus formulations in the treatment of bacterial vaginosis. No improvement in the disease has been noticed
and their use requires further research.

carriers are asymptomatic. Women who were never sexually


active are rarely affected.
Women with bacterial vaginosis are more likely to acquire
other sexually transmitted infections, pregnancy complications,
post-surgery complications and disease recurrence. However,
due to insufficient data, routine screening for these groups is not
recommended.
Diagnosis
Diagnosis of bacterial vaginosis is either clinical or by using the
gram stain. Gram stain is considered by many laboratories as the
golden standard for diagnosis. It is used to estimate the concentration of lactobacilli (Long gram positive rods) and the gram
negative anaerobes.
Nugent scoring system and Amsels clinical criteria are the
most common diagnostic systems used to diagnose bacterial
vaginosis. Nugents scoring system is considered the gold standard for diagnosis. However, it is costly, time consuming and
requires laboratory expertize, to implement. Physicians prefer
Amsels criteria as it is simpler and comparably as effective as
Nugent scoring system. Hay Ison criteria is recommended by the
Bacterial Special Interest group of BASHH to be implemented in
GUM clinics.
A. Clinical criteria (using Amsel) require three of the following
symptoms or signs:
1. Homogeneous, grey-white discharge;
2. Clue cells on wet microscopy;
3. pH of vaginal fluid >4.5;
4. Fishy odour with or without the addition of 10% KOH
(whiff test).
B. Nugent score:
This method estimates the relative proportions of bacterial
types on a Gram stained vaginal smear. A score between
0 and 10 is then assigned.
1. Score of <4 is normal,
2. Score of 4e6 is intermediate
3. Score of >6 is BV.
C. Hay Ison criteria: based on findings on a Gram stained smear
and gives an idea about flora types.
1. Grade 0: Not related to BV, epithelial cells only, no
lactobacilli.
2. Grade 1: (Normal): Lactobacilli predominate
3. Grade 2: (Intermediate): Mixed flora with some Lactobacilli, and Gardnerella or Mobiluncus also present
4. Grade 3 (BV): Few or absent Lactobacilli. Gardnerella
and/or Mobiluncus morphotypes, clue cells,
predominate.
5. Grade 4: Not related to BV, no lactobacilli, Gram ve
cocci only.
Some other tests such as OSOM BV Blue test, Affirm VP III test
and DNA hybridization probe test are comparable to the gram
stain in results. Card tests to detect elevated pH and the cervical
smear tests have low specificity and sensitivity and are not recommended. PCR as a diagnostic tool in BV is still under research.
G. Vaginalis culture is not specific and not recommended.

Pregnancy
In the UK, a BV prevalence of 12% was found in women
attending antenatal care, and 30% in women opting for termination of pregnancy.
Treatment of bacterial vaginosis is recommended for all
symptomatic women. Metronidazole regimen (500 mg) is similar
to non-pregnant population. Using Amstel criteria to define cure,
Yudin MH et al., found that oral metronidazole was as effective
as metronidazole gel for treatment during pregnancy, with a cure
rate of 70%. Metronidazole use during pregnancy is not associated with an increase in congenital malformations. A study by
Ugwumadu A. et al. using gram stain criteria showed an 85%
cure in patients receiving clindamycin. Newer studies have
shown that vaginal clindamycin is safe to administer in pregnancy. Both oral and topical regimens are both as effective and
safe to be prescribed in pregnancy.
Mixed results have been noticed in preterm delivery rate in
patients treated. Harm, no harm and benefit have been found in
several studies. Another study showed a decrease in late
miscarriage and adverse neonatal outcome in the treated group.
However, treatment is recommended for all symptomatic women
with BV as benefits outweigh the risks.
Insufficient data is available for treating asymptomatic pregnant women for which bacterial vaginosis was incidentally
discovered. Therefore, routine screening for bacterial vaginosis
in pregnancy is not recommended.
Metronidazole is secreted in breast milk. Studies have
revealed metronidazole and its metabolites in the plasma of babies, but this level was insignificant a lot lower than the levels
used by the mother to treat the infection. However, some obstetricians recommend deferring breastfeeding for at least 24e48

Treatment
Treatment is recommended for the following group of patients:
 Patients with symptoms.

OBSTETRICS, GYNAECOLOGY AND REPRODUCTIVE MEDICINE 26:11

319

2016 Published by Elsevier Ltd.

REVIEW

hours after completion of treatment. Treatment in breastfeeding


should be with metronidazole twice daily for 5e7 days rather
than a one off dose. Although metronidazole treatment produces
parasitologic cure, certain trials have shown no significant difference in perinatal morbidity following metronidazole
treatment.

B. Complicated vaginal candidiasis is either:


1. Recurrent
2. Severe
3. Albicans/non-albicans candidiasis
4. During pregnancy
5. Immunocompromised conditions
Diagnosis
Clinical diagnosis is important to diagnose vaginal candidiasis.
Symptoms include white discharge, pruritus, burning, dysuria,
pain or redness. Signs include fissure, excoriation, vulval
oedema or thick white discharge. Patients who have some of the
above symptoms and signs can have the following tests to diagnose vulvo-vaginal candidiasis:
 Gram stain or a wet preparation (saline, 10% KOH) of
vaginal discharge will show hyphae, or pseudohyphae or
budding yeasts. 10% KOH will improve visualization and
sensitivity of the test.
 Culture of vaginal discharge will show positive for one of
the yeast species. A culture is indicated if results come
back negative for the Gram or wet stain with persistent
symptoms. Candida isolated from culture with no clinical
symptoms or signs should not be treated, as about 15% of
women harbour candida in their vagina as a commensal.
PCR is not recommended to diagnose candida; culture remains the gold standard for diagnosis.

Management of partner
Routine screening and treatment is not recommended because
studies have shown that treatment of the male partner does not
affect recurrence or relief for women with bacterial vaginosis.
Follow up
It has been found that symptom recurrence occurs within 3e12
months of any regimen used for treatment. Recurrence rate has
been found at 15e30%. No follow up appointments are recommended if symptoms resolve. Patients are advised to contact
their healthcare professional if symptoms persist or recur.
There is limited data for management of persistent or recurring bacterial vaginosis. Using the same regimen or changing the
regimen are both acceptable management plans to treat such
conditions.
Women with recurrent infection may benefit from using
0.75% Metronidazole gel twice weekly for 4e6 months. A study
by Mcclelland and Richardson has found that administering a
monthly dose of 2 g oral metronidazole together with 150 mg
fluconazole is effective in reducing recurrence, although its
suppression is not long term once treatment is discontinued.

Treatment
General advice given to patients:
 Avoid synthetic tight fitting clothes, and avoid washing
underwear in biological washing powder.
 Avoid perfumed local products such as toiletries, antiseptics, douches and wipes.
 Use soap substitutes (e.g. amole root, soap plant root,
soap root bulb, guaiac leaves, papaya leaves and quillaia
bark) and skin conditioners. These products should not be
used more than once per day to clean the vulval area
externally.
 Emollients and skin conditioners may be used several
times per day.
Recommended regimens:
A. Over-the-counter products:
a) Intravaginal creams:
1. Clotrimazole 1% 5 g daily for 7e14 days
2. Clotrimazole 2% 5 g daily for 3 days
3. Miconazole 2% 5 g daily for 7 days
4. Miconazole 4% 5 g daily for 3 days
b) Vaginal suppositories:
1. Miconazole 100 mg daily for 7 days
2. Miconazole 200 mg for 3 days
3. Miconazole 1200 mg one single dose
c) Intravaginal ointment:
1. Tioconazole 6.5%, 5 g single vaginal
application
B. Agents which require a prescription:
a) Intravaginal creams:
1. Butoconazole 2%, 5 g in a single application
2. Terconazole 0.4%, 5 g daily for 7 days
3. Terconazole 0.8%, 5 g daily for 3 days

Vulvovaginal candidiasis (VVC)


Aetiology and classication
Yeast is often found as a commensal organism in the vagina. 10
e20% of women in their reproductive age are colonized by
candida. 75% of women experience symptomatic infection at
least once in their life. Vaginal candidiasis is a condition which
occurs if an alteration in the vaginal environment lead to proliferation of yeast. Hyper-oestrogenic states may lead to vaginal
candidiasis. Some of the hyper-oestrogenic states associated with
candidiasis are pregnancy, diabetes mellitus, obesity, immunosuppression, use of oral contraception and after antibiotic treatment. Tampons, sanitary towels or panty liners are not
associated with VVC when they are used appropriately.
Candida albicans species accounts for about 85% of vaginal
candidiasis. The remaining 15% of vaginal candidiasis is
accounted by Candida krusei, Candida kefyr, Candida tropicalis
and Candida glabrata.
White discharge, pruritus and dysuria are the common
symptoms. These symptoms are non-specific. 75% of patients
with these symptoms will have one episode of vaginal candidiasis. 45% will have two or more episodes.
Vaginal candidiasis is classified into either non-complicated
(85%) or complicated (15%). Below are the criteria used to
classify vaginal candidiasis:
A. Uncomplicated vaginal candidiasis:
1. Infrequent (3 episodes per year)
2. Mild-to-moderate
3. Likely to be Candida albicans
4. Non-immunocompromised women

OBSTETRICS, GYNAECOLOGY AND REPRODUCTIVE MEDICINE 26:11

320

2016 Published by Elsevier Ltd.

REVIEW

b) Vaginal suppositories:
1. Terconazole 80 mg, daily for 3 days
C. Oral agent:
1. Fluconazole 150 mg, single oral dose
All oral azoles and nystatin give about 80% cure rate. If
itching is a major problem, hydrocortisone containing topical
preparations may be of benefit. Creams, suppositories and ointments in the above preparations may weaken latex condoms and
diaphragms. Condom product information should be reviewed.

T. vaginalis
Prevalence
Trichomoniasis is usually acquired through sexual contact. The
prevalence of the disease among black women is 13%, while its
prevalence among white is estimated at a rate of about 1.8%. It
affects about 11% of women aged more than 40 years. The
flagellate protozoan, T. vaginalis, is responsible for the disease.
Abnormal vaginal discharge, pruritus and dysuria are the main
symptoms. The characteristic feature of the infection is the
strawberry cervix. This appears as a friable, erythematous cervix
with punctate areas of exudate.
Urethral infection is present in 90% of cases. However,
isolating the protozoan from just the urethra is just below 5% of
cases.
Screening for T. vaginalis in asymptomatic women is not
recommended. Screening can be considered in asymptomatic
women who are at high risk for infection. These include patients
with multiple sexual partners, drug abusers and patients with a
history of STIs. However, there is insufficient data regarding
screening asymptomatic women without additional risk factors.

Pregnancy
Vulvovaginal candidiasis is common among pregnant women.
Up to 40% of carriers will have symptoms during pregnancy.
Only topical azole therapy is recommended for symptomatic
pregnant women. Animal studies have found that Oral antifungals triazoles (fluconazole and itraconazole) causes fetal abnormalities and should be avoided during pregnancy.
HIV
Vaginal Candida colonization rates among women with HIV
infection are higher than among seronegative women with
similar demographic and risk behaviour characteristics, and the
colonization rates correlate with increasing severity of immunosuppression. Symptomatic VVC is also more frequent in
women with HIV infection and similarly correlates with severity
of immunodeficiency. In addition, among women with HIV
infection, systemic azole exposure is associated with the isolation of non-albicans Candida species from the vagina.
On the basis of available data, therapy for uncomplicated and
complicated VVC in women with HIV infection should not differ
from that for seronegative women. Long-term prophylactic
therapy with fluconazole at a dose of 200 mg weekly has been
effective in reducing C. albicans colonization and symptomatic
VVC.

Diagnosis
About 70e85% of infected patients are asymptomatic. Increased
local polymorph nuclear leucocytes (PMNL) is the main host
response to infection.
Sites for sampling are either by high vaginal swabs from the
posterior fornix, self-taken vaginal swabs or urine samples.
Methods for detection of T. vaginalis include:
 NAAT (nucleic acid amplification testing) is a highly sensitive method for detection of T. Vaginalis. It can detect
three to five times more than wet preparations using direct
microscopy. It is now considered the gold standard for
detection of T. vaginalis. Specimens are usually retrieved
from the vagina, endocervix or urine. This method detects
the protozoal RNA with a high sensitivity of 95.3e100%
and a high specificity of 95.2e100%.
 Direct microscopy: this is the method of choice for
screening. A wet preparation has a low sensitivity of 50
e70%. Vaginal discharge mixed with a drop of saline on a
glass slide can be used to detect T. vaginalis motility. T.
vaginalis lose their motility quickly, therefore the wet
preparation should be read within 10 minutes of collection. Microscopy with a magnification of 400 is used to
confirm the morphology and visualize the flagella. Acridine orange when used as a stain for films was found
more sensitive than unstained wet preparations. This
method is more practical due to its lower cost compared
to NAAT.
 Culture was considered in the past as the gold standard for
detection of the protozoan. It has a sensitivity of 75e96%
and a specificity of up to 100%. For culture, vaginal
discharge is preferred compared to a urine specimen, as
urine culture is less specific.
 Immunomodulation: OSOM Trichomonas Rapid Test and
the Affirm VP III.
 Cervical smear Pap tests can incidentally detect T. vaginalis, but their false positive and false negative rates make
them unreliable as a diagnostic tool

Management of partner
Asymptomatic partners do not require testing or treatment; as
vaginal candidiasis is not a sexually transmitted disease. However, few male partners may benefit from topical antifungal
agents if they experience balanitis (pruritus with erythema of the
penile glans).
Follow up
No follow-up is required for a single episode of vaginal candidiasis. However, if symptoms persist or recur within 2 months of
over-the-counter products, a medical evaluation and investigations are essential.
Recurrent vulvo-vaginal candidiasis
This is defined as four or more episodes of candidiasis within a
year. Prevalence within symptomatic women is expected to be
less than 5%. Non-albicans yeast species cause 10e20% of
recurrent disease. A comprehensive history and exclusion of risk
factors is mandatory for management. Treatment options are a
longer duration of use (7e14 days) of vaginal creams and suppositories, or using oral Fluconazole weekly for 6 months as
suppression and maintenance regimen. Most patients are disease
free during treatment and for around 6 months afterwards but
most do relapse within a year.

OBSTETRICS, GYNAECOLOGY AND REPRODUCTIVE MEDICINE 26:11

321

2016 Published by Elsevier Ltd.

REVIEW

Due to limited data regarding the safety of tinidazole use in


pregnancy, it is better avoided. Breastfeeding should be deferred
for at least 72 hours following the uptake of the 2 g tinidazole
dose.

Treatment
Indications for treatment:
 Testing positive for T. vaginalis, regardless of symptoms
 Treatment of sexual partners
Recommended regimen:
 A single dose of 2 g metronidazole orally
OR
 A single dose of 2 g tinidazole orally
Alternative regimen
 Twice daily 500 mg metronidazole orally for 7 days
The only effective treatment against T. vaginalis are metronidazoles. The cure rate for metronidazole is 84e98%, while
that for tinidazole, a more expensive drug, is 92e100% Metronidazole gel is less effective due to its low absorption rate and is
not recommended for treatment.
Alcohol, sexual intercourse or vaginal douching may hinder
the effectiveness of treatment. Alcohol should not be consumed
during treatment up to 24 hours after completion of a course of
metronidazole, or 72 hours after completion of a tinidazole
course, as it may precipitate a disulfiram-like reaction. Sexual
intercourse should be avoided during and at least for one week
after treatment. Douching is not recommended as it alters the
vaginal flora and increases the risk of infection.

HIV
There is increased transmission of both HIV and TV as a result of
increased shedding of the virus. Patients with HIV should be
given a 7-day course of metronidazole so as to ensure full
treatment of the infection. Retesting after 3 months may be
warranted in some cases.
Management of partner
Infection is readily transmitted between partners by the act of
unprotected sexual intercourse. Condom use is the main method
for prevention against infection. Uncircumcised men carry
greater risk of transmission to their partners than those who are
circumcised.
Treatment of the partner is recommended to prevent reinfection or transmission among partners. Abstaining from sexual
intercourse is imperative until both partners are treated. The
same regimen of treatment is advised for the partner. Treatment
should be commenced regardless of their test results.
Follow up
Peterman found a recurrence rate of 17% after treatment for T.
Vaginalis. A test of cure is recommended after 3 months of
completing the treatment course. Retesting by NAAT can be done
after 2 weeks of treatment. Limited data is available to guide
retesting the male partner.

Persistent or recurrent T. Vaginalis


Persistent or recurrent Trichomoniasis can be a result of reinfection by the partner or antibiotic-resistant T. Vaginalis
strains. 4e10% of T. vaginalis are resistant to metronidazole and
1% are resistant to tinidazole. Avoid single dose therapy for
treatment of persistent or re-infection with T. Vaginalis.
If the single dose of 2 g metronidazole fails, a 500 mg
metronidazole twice daily oral dose for 7 days is recommended.
If this management fails, then 2 g metronidazole or 2 g tinidazole
for 7 days is advisable. Susceptibility to metronidazole or tinidazole should be tested if there is no cure after 1 week of treatment. Intravaginal tinidazole, plus high dose 2e3 g daily oral
tinidazole, could be considered for highly resistant strains.
However, expert opinion should be sought.

Desquamated inammatory vaginitis (DIV)


This is an uncommon form of chronic purulent vaginitis, which
mainly occurs in Caucasians with a peak occurrence in perimenopause. Its aetiology is unknown and it is a diagnosis of
exclusion. Its main symptoms are purulent discharge and
vestibular-vaginal irritation and dyspareunia. It has an estimated
incidence of 0.8e4.3% of women referred with persistent vaginal
discharge. The symptoms and signs are non-specific and extensive investigation may be required.

Pregnancy
Screening in pregnancy for T. vaginalis in asymptomatic women
is not recommended. T. vaginalis is a risk factor for the vertical
transmission of HIV. Therefore, screening and treatment of T.
vaginalis is recommended for HIV positive pregnant women.
These individuals should have a repeat test 3 months after
treatment.
Infection in pregnancy is associated with a two-to-three-fold
increase in rates of preterm delivery and low birth weight. The
recommended treatment in pregnancy is 2 g oral metronidazole
as a single dose. Treatment may prevent genital or respiratory
infection in the new born.
Metronidazole can be prescribed at any gestational age, if
indicated. Although it crosses the placenta, no evidence of teratogenicity has been established.
Metronidazole is secreted in breast milk but only low plasma
levels are found in the breast fed babies of women using this
medication. However, some obstetricians recommend deferring
breastfeeding for at least 24e48 hours after completion of
treatment.

OBSTETRICS, GYNAECOLOGY AND REPRODUCTIVE MEDICINE 26:11

Presentation
Most patients with DIV will have complaints for more than a
year. Most patients are typically symptomatic, although asymptomatic DIV occasionally occurs. Purulent vaginal discharge
ranges from mild to profuse, associated with signs of inflammation, such as focal petechiae (30e70%) or diffuse vaginal
erythema. Up to 27% of patients may have a similar lesion
involving the cervix and biopsy may show dense lymphocytic
infiltrates.
There is a marked increase in inflammatory cells, predominantly polymorphonuclear leukocytes, together with an increase
in immature squamous cells. Vaginal flora is abnormal with the
loss of lactobacillus morphotype and pH is always elevated
above 4.5.
The diagnosis of DIV is by exclusion of other conditions as
listed:
 Infectious diseases: T. vaginalis, Group A Streptococcus,
Cervicitis

322

2016 Published by Elsevier Ltd.

REVIEW

 Immune/autoimmune induced inflammation: erosive


lichen planus, cutaneous vesiculobullous disorder
 Hormonal conditions: oestrogen deficiency (atrophic
vaginitis)
 Contact dermatitis: chemical vaginitis, allergy (latex,
sperm)
 Miscellaneous: trauma, foreign body, urinary incontinence
including vesico-vaginal fistula, or cervical tumour,

is not recommended, except in special circumstances. DIV is an


uncommon severe form of vaginitis and should be considered
among women where symptoms have persisted for a long time
despite multiple pharmaceutical treatments. Examination, history and investigations are all complementary in the diagnosis
and the management process to prevent over treatment or under
diagnosis of susceptible women.
A

Management
It has now been accepted that a trial of treatment with topical
oestrogens is a useful method to differentiate between DIV and
atrophic vaginitis. Menopausal women should continue to apply
vaginal oestrogens in addition to the anti-inflammatory treatment
for DIV as well. It is reasonable to send a vaginal culture in
search for Group A Streptococcus and if positive, be treated. Both
topical vaginal clindamycin and local vaginal corticosteroids are
useful and sometimes curative as both have anti-inflammatory
effects.
The recommended regimens are as follows:
 Clindamycin:
 Insert 2% clindamycin (5 g) vaginally once a night for
three weeks, followed by a maintenance therapy twice a
week for two months
 Clindamycin 200 mg vaginally each night for three
weeks, followed by a maintenance therapy twice a week
for two months
 Corticosteroids:
 Vaginal hydrocortisone 200e500 mg once a night for
three weeks, followed by a maintenance therapy twice a
week for two months
 Vaginal cortisone acetate suppository 25 mg twice a day
for three weeks, followed by polymorphonuclear leukocytes maintenance therapy three times a week for two
months.
25% of patient will develop a symptomatic yeast infection. It
is recommended to add oral fluconazole to the treatment regimen
in patients prone to develop such an infection.
Women with DIV require long term follow up to assess
improvement and remission. The evaluation includes measurement of inflammatory and parabasal cell numbers, vaginal pH
and the presence of normal healthy lactobacillus.

FURTHER READING
British Association of Sexual Health and HIV Infection. UK national
guideline on the management of vulval conditions 2014.
Guidelines developed by the Clinical Effectiveness Unit (CEU) of the
Faculty of Sexual and Reproductive Health Care (FSRH) in
collaboration with the British Association for Sexual Health and HIV
(BASHH) [FSRH, 2012].
Gutman RE, Peipert JF, Weitzen S, Blume J. Evaluation of clinical
methods for diagnosing bacterial vaginosis. Obstet Gynecol 2005;
105: 551e6.
Larsson P-G. Treatment of bacterial vaginosis. Int J STD AIDS 1992; 3:
239e47.
National Institute for Health and Care Excellence Clinical Knowledge
Summary. Vaginal Discharge. May 2013.
Peterman TA, Tian LH, Metcalf CA, et al. High incidence of new
sexually transmitted infections in the year following a sexually
transmitted infection: a case for rescreening. Ann Intern Med 2006;
145: 564e72.
Reichman O, Sobel J. Desquamated inammatory vaginitis (in) vulvovaginal disease-best practice & research. Clin Obstet Gynaecol
2014; 28: 1042e50. Elsevier, Newyork.
Sherrard J, Donders G, White D, Jensen JS. European (IUSTI/WHO)
guideline on the management of vaginal discharge. 2011, www.
isuti.org.
Yudin MH, Landers DV, Meyn L, et al. Clinical and cervical cytokine
response to treatment with oral or vaginal metronidazole for bacterial vaginosis during pregnancy: a randomized trial. ObstetGynecol 2003; 102: 527e34.

Practice points
C

Conclusion
Management of vaginal discharge could never be accomplished
without a thorough medical history. pH strips are a simple
additional diagnostic test that is underutilized in gynaecological
clinics. Routine testing or screening of all gynaecological patients

OBSTETRICS, GYNAECOLOGY AND REPRODUCTIVE MEDICINE 26:11

323

Vaginal discharge is a common complaint e not all vaginal


discharge is abnormal or requires treatment
A thorough history should yield valuable information. pH testing
and swabs can be taken as necessary
Extra caution must be exhibited when managing pregnant women
with pathological vaginal discharge

2016 Published by Elsevier Ltd.

REVIEW

Contraception in women
with medical conditions

mortality. These risks are usually greater than the risks


associated with contraceptive use. Effective contraception
should not be withheld without good cause.
 some contraceptives can cause deterioration in certain
medical conditions.
 medical conditions and their treatments may reduce the
effectiveness of some methods of contraception.
 medical treatments may have teratogenic effects.
In this review we will consider the effectiveness of currently
available methods of contraception and their suitability in
women who have diseases of the cardiovascular, gastrointestinal, reproductive and immune systems using the most up to
date evidence. We will also consider contraception in obesity.
Abortion is clearly important in women with medical conditions
who have unplanned pregnancy and referral to a maternal
medicine specialist for counselling may be appropriate to discuss
the risks of continuing with the pregnancy and termination.
The UK medical eligibility criteria for contraceptive use (UK
MEC) will be referred to throughout this review. The UK MEC
gives professionals guidance on the safety of contraception in
women with medical conditions and other characteristics.
 UK MEC category 1 A condition for which there is no
restriction for the use of the method.
 UKMEC category 2 A condition where the advantages of
using the method generally outweigh the theoretical or
proven risks.
 UKMEC category 3 A condition where the theoretical or
proven risks usually outweigh the advantages of using the
method. The provision of a method requires expert clinical
judgment and/or referral to a specialist contraceptive
provider, since use of the method is not usually recommended unless other more appropriate methods are not
available or not acceptable.
 UKMEC category 4 A condition which represents an
unacceptable health risk if the method is used.

Sarah L Millar
Sharon T Cameron

Abstract
Pregnancy can result in both maternal and fetal morbidity in women
with medical conditions, making the provision of contraception a
crucial part of their care. Obstetricians and gynaecologists will
frequently encounter these women in the outpatient and inpatient settings and will be expected to have knowledge of the safest and most
effective methods of contraception for that individual. The UK medical
eligibility criteria gives guidance regarding the safety of contraceptive
methods in women with medical conditions. This review will explain
the rationale behind this guidance for women with conditions of the
cardiovascular, gastrointestinal, reproductive and immune systems
as well as considering the effectiveness of currently available methods
of contraception.

Keywords contraception; medical eligibility; pregnancy; safety

Introduction
Forty six percent of pregnancies in the UK are unplanned. Unplanned pregnancies can have far reaching consequences for
women and healthcare services. It is estimated that unintended
pregnancy costs the NHS 1 billion pounds annually. Increasing
the uptake of effective contraception is one way of reducing
unplanned pregnancies. It is thought that 11 is saved in
healthcare costs for every 1 spent on contraception in England.
The physiological changes in pregnancy may cause some
medical conditions to deteriorate in pregnancy (Table 1). Also,
some medical conditions or their treatments, may adversely
affect the outcome of pregnancy. Pre pregnancy counselling is
often imperative in women with chronic health conditions and
women may be advised to delay or avoid pregnancy. These
women need reliable and effective contraception.
However, clinicians may be unfamiliar with which methods of
contraception are suitable for such women and may be concerned about the safety of contraception. There are several issues
to consider when faced with a woman of reproductive age who
does wish to become pregnant and has an acute or chronic
medical condition:
 pregnancy in women with co-existing medical conditions
can result in maternal and perinatal morbidity and

Efcacy and mode of action of currently available methods


of contraception
The modes of action and effectiveness of currently available
contraception in the UK are outlined in Table 2. Long acting
reversible contraception (LARC) progestogen only implant, intrauterine system (IUS), intrauterine device (IUD) and depot
medroxyprogesterone acetate (DMPA) have lower failure rates, are
acceptable to women and have been shown to be more cost effective at 1 year than contraceptive pills. The effectiveness of LARC
methods generally do not rely on user compliance and so their
typical failure rates are far superior to non LARC methods. Therefore, all women, and in particular those with co-existing medical
conditions, should be encouraged to use LARC where appropriate.

Cardiovascular system
Sarah L Millar MBCHB MFSRH Specialty Trainee Community Sexual
and Reproductive Health, Chalmers Sexual Health Clinic, Edinburgh,
Scotland, UK. Conicts of interest: none declared.

Cardiac disease is the leading cause of maternal death in the UK


from indirect causes (deaths resulting from previous existing
disease which was aggravated by the physiological effects of
pregnancy). Delaying or avoiding pregnancy in severe disease
may be advised. Although there are some concerns regarding
safety of some forms of contraception in women with CVD, the
risks are generally much smaller than those of pregnancy. For

Sharon T Cameron MBCHB MD MFSRH FRCOG Chalmers Sexual Health


Clinic, Edinburgh, Scotland, UK; Consultant Gynaecologist and
Clinical Reader, Obstetrics and Gynaecology, University of
Edinburgh, Edinburgh, UK. Conicts of interest: SC holds an
independent investigator initiated research grant from Pzer UK.

OBSTETRICS, GYNAECOLOGY AND REPRODUCTIVE MEDICINE 26:11

324

2016 Published by Elsevier Ltd.

REVIEW

example, the risk of venous thromboembolism (VTE) in the


postpartum period is 300e400 per 10,000 women compared with
5e12 per 10,000 women using combined hormonal contraception (CHC). Table 3 outlines the UK MEC guidance for cardiac
diseases.

Conditions that may pose a significant health risk during


pregnancy
C

C
C
C
C
C

C
C

C
C
C
C

Bariatric surgery within the


last 2 years
Breast cancer
Cardiomyopathy
Complicated valvular disease
Cystic fibrosis
Diabetes: insulin dependent, or
with nephropathy/retinopathy/
neuropathy or other vascular
disease
Ischaemic heart disease
Malignant liver tumours
(hepatocellular carcinoma)
Morbid obesity (BMI 40 kg/m2)
Organ transplant/failure
Rheumatoid arthritis
Severe (decompensated)
cirrhosis

C
C

C
C
C
C
C

C
C
C
C

Endometrial or ovarian
cancer
Epilepsy
Gestational trophoblastic
neoplasia
HIV-related disease
Hypertension
Sickle cell disease
Stroke
Systemic lupus
erythematosus (SLE)
Systemic sclerosis
Thrombogenic conditions
Tuberculosis
Teratogenic drugs

Combined hormonal contraception


The risks of ischaemic heart disease and stroke have been shown
to be increased in women using CHC who have hypertension
and/or smoke. CHC can also elevate blood pressure and cause
fluid retention. Therefore, the risks of prescribing CHC usually
outweighs the benefits in women who have existing cardiac
disease (i.e. cardiomyopathy, atrial fibrillation, complicated
valvular or congenital heart disease) or significant risk factors for
cardiovascular disease. Safer and more effective methods should
be offered.
It is well established that CHC users are at in increased risk of
VTE compared with non CHC users. Third generation pills
appear to have the highest risk of VTE. Progestogens modify the
thrombogenic effects of oestrogens in CHC and this explains the
differences seen in VTE rates between pills. However, it should
be remembered that the absolute risk of VTE is still small especially when compared to the risk during pregnancy or
postpartum.
Women with thrombogenic mutations (i.e. factor V Leiden,
prothrombin mutation, protein S, protein C and antithrombin

Reproduced under licence from FSRH. Copyright Faculty of Sexual and


Reproductive Healthcare 2006e2016.

Table 1

Mode of action and effectiveness of contraception


Method

Primary mode of action

Typical use failure (% of women


experiencing an unintended
pregnancy in the first
year of use)

Perfect use failure (% of


women experiencing an
unintended pregnancy
in the first year of use)

Progestogen only implant


Sterilisation
Male
Female
Intrauterine contraception
IUS

Ovulation inhibition

0.05

0.05

Division or occlusion vas deferens


Tubal occlusion

0.15
0.5

0.15
0.5

Changes to endometrium and


cervical mucus prevent
fertilization and implantation
Copper toxic to sperm
Ovulation inhibition
Ovulation inhibition

0.2

0.2

0.8
6
9

0.6
0.2
0.3

IUD
Depo-provera
Combined hormonal
contraception (pills, patch, ring)
Progestogen only pill

Diaphragm
Condom
Male
Female

Ovulation inhibition
(desogestrel pills)
Cervical mucus changes
(traditional pills)
Barrier

0.3

12

Barrier
Barrier

18
21

2
5

Table 2

OBSTETRICS, GYNAECOLOGY AND REPRODUCTIVE MEDICINE 26:11

325

2016 Published by Elsevier Ltd.

REVIEW

UK MEC categorization for women with cardiovascular disease


Condition
Multiple risk factors for cardiovascular disease
(such as smoking, diabetes, hypertension,
obesity and dyslipidaemias)
Adequately controlled hypertension
Hypertension:
Systolic >140e159 mmHg or diastolic >90
e99 mmHg
Hypertension:
Systolic 160 mmHg or diastolic 100 mmHg
Vascular disease
Current and history of ischaemic heart disease
Stroke (history of cerebrovascular accident,
including TIA)
Cardiomyopathy with normal cardiac function
Cardiomyopathy with impaired cardiac
function
Atrial fibrillation
Known long QT syndrome
Uncomplicated valvular and congenital heart
disease
Complicated valvular and congenital heart
disease (e.g. pulmonary hypertension, history
of subacute bacterial and endocarditis)
History of VTE
Current VTE (on anticoagulants)

Cu-IUD

LNG-IUS

IMP

DMPA

POP

CHC

1
1

1
1

1
1

2
1

1
1

3
3

1
1

3
3

2
I
2
2

2
I
2
2

1
2

1
2

1
2

1
2

1
2

2
4

2
1

2
2

2
1

4
2

1
I
3
1

C
1

2
I
3
1

C
3
3

C
1

2
I
2
2

C
3
3

C
3
3

4
4
4

1
1

2
2

2
2

2
2

2
2

4
4

Cu-IUD copper intrauterine device, LNG-IUS levonorgestrel intrauterine system, IMP implanon/nexplanon, DMPA depot medroxyprogesterone acetate (IM and
SC), POP progestogen only pill, CHC combined hormonal contraception, VTE venous thromboembolism, I initiation of a method, C continuation of a method.
Reproduced under licence from FSRH. Copyright Faculty of Sexual and Reproductive Healthcare 2006e2016.

Table 3

and vasovagal episodes. Women with cardiac disease who would


be at risk from bradycardia or vasovagal episodes should have
their IUC fitted in a hospital setting.
Prophylactic antibiotic cover to prevent endocarditis is no
longer advised in women with artificial heart valves or previous
endocarditis.
The risk of bleeding during IUC insertion in women taking
aspirin or warfarin does not appear to be significantly increased.
The FSRH advise that women who have an INR within the therapeutic target range have their IUC fitted in the community without
interruption to their anticoagulant. This guidance was published
prior to the common use of new direct oral anticoagulants (DOACs)
such as apixaban but the guidance is likely to be the same.
Severity of CHD will vary between individuals and clinicians
should seek advice from cardiology if required. In general, LARC
methods would be most suitable in women with cardiovascular
disease because of their superior efficacy and overall safety in
women.

deficiencies) who use CHC have a significantly higher risk (up to


twenty fold) of VTE than those who do not use CHC. Women
who have current or past VTE or significant risk factors for VTE
should not use CHC.
Progestogen only contraception
Progestogen only contraception (POC) does not appear to be
associated with an increased risk of venous or arterial thrombosis. However, there is some evidence that DMPA (which is a
higher dose of progestogen and has a longer half-life compared to
other progestogens) causes changes in lipid metabolism which
could confer increased risk of atherosclerosis. There is no substantial evidence, however, that DMPA increases CVD or VTE in
clinical practice.
Intrauterine contraception
Instrumentation of the cervix during intrauterine contraception
(IUC) insertion can sometimes be associated with bradycardia

OBSTETRICS, GYNAECOLOGY AND REPRODUCTIVE MEDICINE 26:11

326

2016 Published by Elsevier Ltd.

REVIEW

use the most effective non oral methods of contraception such as


the progestogen implant, IUD or IUS.

Gastrointestinal system
Inammatory bowel disease
Inflammatory bowel disease (IBD) (Crohns disease and ulcerative colitis) most commonly presents in women of reproductive
age. Women should be advised that there is no good evidence
that contraception causes or exacerbates IBD and the UK MEC
guidance is summarized in Table 4. Women with IBD are
generally advised to avoid pregnancy during acute exacerbations
and whilst taking teratogenic medications. Clinicians may wish
to refer to the UKTIS website for accurate and up to date information on the risks of teratogenicity with medications (www.
uktis.org).
IBD can result in malabsorption if the small bowel is affected
or there has been small bowel resection. In these women the
efficacy of oral contraception is likely to be reduced and non oral
methods (progestogen implant, IUD or IUS) are preferable.
Women with IBD have been shown to have slightly reduced
bone mineral density (BMD) and modestly increased risk of
fracture compared with women without IBD. This is probably
related to lower body mass index (BMI) and corticosteroid use.
Reductions in BMD with DMPA (which resolve on stopping)
have also been observed. If DMPA is considered the most suitable method for the woman then she should be advised on lifestyle factors to maintain bone health (stopping smoking,
exercise, adequate calcium and vitamin D intake) and her suitability for DMPA reviewed every 2 years.
Although women with IBD appear to be at slightly increased
risk of VTE there are no restrictions to the use of CHC in women
with mild disease. However, the risk of VTE may be raised to an
unacceptable level in women with IBD who have additional risk
factors for VTE such as prolonged immobilization after surgery
or severe disease. The risks of using CHC in these situations may
outweigh the benefits.
Contraceptive choice in women with IBD should be individualized and dependent on the severity of her symptoms, the
necessity for effective contraception and the risk of complications
such as VTE and osteoporosis. Ideally women with IBD should

Liver disease
The UK MEC for women with liver disease is shown in Table 5.
Hormonal contraception is metabolized by the liver and can
adversely affect liver function in women with pre-existing liver
disease. However, hormonal contraceptives are unlikely to cause
significant damage in women with mild chronic hepatitis, mild
cirrhosis or in carriers of hepatitis and so their use should not be
restricted.
The liver has oestrogen receptors but not progesterone receptors making it slightly more susceptible to damage from
oestrogen containing contraceptives. This is reflected in the difference in the UK MEC categorization for CHC and progestogen
only methods. The effects of ethinylestradiol are no less if given
via a patch or ring.

Reproductive system and breast


Overall women who use contraception are not at increased risk
of cancer compared to women who use no contraception.
Furthermore, the risk of ovarian, endometrial and large bowel
cancer is significantly reduced in women using CHC. The Royal
College of General Practitioners oral contraception study
included more than a million women years of observation of
women using the oral contraceptive pill (mainly cocp) and reported that users had a 50% reduced risk of ovarian and endometrial cancer after 8 years of use compared to non users, and
that this protective effect persisted for at least 15 years. However,
the same study, found that the risk of invasive cervical cancer
and carcinoma in situ was increased in CHC users and this risk
increased with duration of use and the presence of persisting
HPV disease. The collaborative group in hormonal factors in
breast cancer, which included 53,297 women with breast cancer
and 100,239 women without breast cancer, found that there was
a small increased risk of breast cancer in current CHC users (RR
1.24) but this risk disappeared after 10 years of stopping CHC.

UK MEC categorization for women with liver disease


Condition

Cu-IUD

LNG-IUS

IMP

DMPA

POP

Viral hepatitis: acute or flare

Carrier of hepatitis
Chronic hepatitis
Mild cirrhosis
Severe (decompensated) cirrhosis
Benign focal nodular hyperplasia
Benign hepatocellular adenoma
Malignant hepatocellular carcinoma

1
1
1
1
1
1
1

1
1
1
3
2
3
3

1
1
1
3
2
3
3

1
1
1
3
2
3
3

1
1
1
3
2
3
3

CHC
I
3
1
1
1
4
2
4
4

C
1

Cu-IUD copper intrauterine device, LNG-IUS levonorgestrel intrauterine system, IMP implanon/nexplanon, DMPA depot medroxyprogesterone acetate (IM and
SC), POP progestogen only pill, CHC combined hormonal contraception, VTE venous thromboembolism, I initiation of a method, C continuation of a method.
Reproduced under licence from FSRH. Copyright Faculty of Sexual and Reproductive Healthcare 2006e2016.

Table 4

OBSTETRICS, GYNAECOLOGY AND REPRODUCTIVE MEDICINE 26:11

327

2016 Published by Elsevier Ltd.

REVIEW

UK MEC categorization for women with gynaecological cancers


Condition
Cervical intraepithelial neoplasia (CIN)
Cervical cancer awaiting treatment
Cervical cancer: radical trachelectomy
Ovarian cancer
Endometrial cancer

Cu-IUD
1
I
4
3
1
I
4

LNG-IUS

C
2

C
2

2
I
4
3
1
I
4

C
2

C
2

IMP

DMPA

POP

CHC

1
2

2
2

1
1

2
2

2
1
1

2
1
1

1
1
1

2
1
1

Cu-IUD copper intrauterine device, LNG-IUS levonorgestrel intrauterine system, IMP implanon/nexplanon, DMPA depot medroxyprogesterone acetate (IM and
SC), POP progestogen only pill, CHC combined hormonal contraception, I initiation of a method, C continuation of a method.
Reproduced under licence from FSRH. Copyright Faculty of Sexual and Reproductive Healthcare 2006e2016.

Table 5

hydatidiform mole or for 1 year after chemotherapy. Contraception therefore needs to be discussed with women. A recent
large (over 2000 women) historical database review found that
women with complete hydatidiform mole who used hormonal
contraception while their HCG was elevated from complete
hydatidiform mole had no difference in time to remission of
their HCG levels or rates of progression to neoplasia compared
with women who did not use hormonal contraception. There is
also some evidence that using CHC does not adversely affect
the regression of neoplasia in women undergoing chemotherapy. The UK MEC guidance relating to GTD and IUC reflects
the theoretical concern of increased risk of perforation when
molar tissue is still present. The UK MEC guidance is summarized in Table 6.

Gynaecological cancer
The management of gynaecological cancers will usually result in
sterility (i.e. pelvic clearance, chemotherapy). Those who have
fertility sparing treatment, by definition, will not require
contraception. However, contraception is important in the lead
up to treatment. Women generally remain fertile while awaiting
treatment and an unplanned pregnancy may delay or alter her
treatment and cause emotional distress.
The benefits of using the progestogen only implant, POP, DMPA
or CHC generally outweigh the risks in women with gynaecological
cancer (Table 6). Anatomy may be distorted in women who have
had trachelectomy and IUC should generally be avoided unless no
other method is acceptable. There may be an increased risk of
bleeding and infection if IUC is fitted in women who are awaiting
treatment for cervical or endometrial cancer. However, if the
women already have IUC, it does not need to be removed.

Breast cancer
Breast cancer treatments (chemotherapy, anti oestrogens) do not
usually result in permanent, or even temporary, sterility and so
contraception is still relevant. Breast cancer is hormone sensitive
and the use of hormonal contraception (including the LNG-IUS)
should be avoided in women with current or past breast cancer
because of the risk of acceleration or recurrence of disease. The
copper IUD would a good option for women with current or past
breast cancer.

Gestational trophoblastic disease


Gestational trophoblastic disease (GTD) includes women with
partial or complete hydatidiform mole and gestational trophoblastic neoplasia. Surgical evacuation of the uterus is the
treatment of choice for hydatidiform mole and for trophoblastic
neoplasia it is chemotherapy. Women are generally advised not
to conceive until follow up has been completed for

UK MEC categorization for women with gestational trophoblastic disease


Condition

Cu-IUD

LNG-IUS

IMP

DMPA

POP

CHC

Gestational trophoblastic disease:


undetectable hCG levels
Gestational trophoblastic disease: decreasing
hCG levels
Persistently elevated hCG levels or malignant
disease

Cu-IUD copper intrauterine device, LNG-IUS levonorgestrel intrauterine system, IMP implanon/nexplanon, DMPA depot medroxyprogesterone acetate (IM and
SC), POP progestogen only pill, CHC combined hormonal contraception.
Reproduced under licence from FSRH. Copyright Faculty of Sexual and Reproductive Healthcare 2006e2016.

Table 6

OBSTETRICS, GYNAECOLOGY AND REPRODUCTIVE MEDICINE 26:11

328

2016 Published by Elsevier Ltd.

REVIEW

UK MEC categorization for women with breast disease


Condition

Cu-IUD

LNG-IUS

IMP

DMPA

POP

CHC

Current breast cancer


Past breast cancer
Family history of breast cancer
Carriers of known gene mutations associated
with breast cancer (e.g. BRCA 1/BRCA 2)

1
1
1
1

4
3
1
2

4
3
1
2

4
3
1
2

4
3
1
2

4
3
1
3

Cu-IUD copper intrauterine device, LNG-IUS levonorgestrel intrauterine system, IMP implanon/nexplanon, DMPA depot medroxyprogesterone acetate (IM and
SC), POP progestogen only pill, CHC combined hormonal contraception, VTE venous thromboembolism, I initiation of a method, C continuation of a method.
Reproduced under licence from FSRH. Copyright Faculty of Sexual and Reproductive Healthcare 2006e2016.

Table 7

Enzyme inducing antiepileptic drugs (EIAEDs) include


phenobarbital, primidone, phenytoin, carbamazepine, oxcarbazepine, felbamate and topiramate. EIAEDs increase the clearance
of oestrogen and progestogen in the liver and hence the efficacy
of CHC, the POP and progestogen only implant (implanon/nexplanon) may be reduced. Women using EIAEDs should be
encouraged to use DMPA, IUS or IUD as the efficacy of these
methods do not appear to be altered by the use of EIAEDs and are
more effective than condoms alone.
It is now recognized that CHC can reduce lamotrigine levels
and worsens seizure control through the induction of glucuronidation. Women should be encouraged to use progestogen
only methods or the IUD as these do not appear to affect lamotrigine levels.

Women are often concerned about using hormonal contraception if they have a family history of breast cancer. From the
limited data that is available, hormonal contraception does not
appear to significantly increase the risk of breast cancer in
women who have a family history of breast cancer (see Table 7).
Benign gynaecological disease
Hormonal contraception is often used in the treatment of menstrual disturbance due to its favourable effects on dysmenorrhoea and heavy menstrual bleeding. However, in conditions
where the uterine cavity is significantly distorted or there is a risk
of introducing infection into the pelvis, insertion of IUC may not
be appropriate.

Neurological diseases
Migraine
Migraine is a common complaint among women of reproductive
age. Ischaemic stroke is more common in women with migraine
with aura compared to those without aura. The risk increases
further (two to four fold) if a woman with migraine with aura
takes CHC. The UK MEC (Table 8), therefore, advises against the
use of CHC in women who have migraine with aura. POC does
not appear to increase the risk of stroke and can therefore be
used in women with migraine with aura.

Epilepsy
It is important that women with epilepsy plan their pregnancy in
order to reduce the risk of teratogenicity from antiepileptic drugs
(AEDs) (in particular sodium valproate) and optimize seizure
control. There are no concerns over safety of contraceptives in
women with epilepsy. However, some AEDs can reduce the efficacy of contraception and some contraceptives alter the efficacy
of AEDs.

UK MEC categorization for women with neurological disease


Condition

Cu-IUD

LNG-IUS

IMP

DMPA

POP

Non-migrainous headache

Migraine without aura at any age

Migraine with aura at any age


History (>5 year ago) of migraine with aura at
any age

1
1

2
2

2
2

2
2

I
1
2
2

CHC

C
2

I
1
I
2
4
3

C
2
C
3

Cu-IUD copper intrauterine device, LNG-IUS levonorgestrel intrauterine system, IMP implanon/nexplanon, DMPA depot medroxyprogesterone acetate (IM and
SC), POP progestogen only pill, CHC combined hormonal contraception, VTE venous thromboembolism, I initiation of a method, C continuation of a method.
Reproduced under licence from FSRH. Copyright Faculty of Sexual and Reproductive Healthcare 2006e2016.

Table 8

OBSTETRICS, GYNAECOLOGY AND REPRODUCTIVE MEDICINE 26:11

329

2016 Published by Elsevier Ltd.

REVIEW

UK MEC categorization for women with immune system disease


Condition

Cu-IUD

LNG-IUS

IMP

DMPA

POP

CHC

Rheumatoid arthritis
SLE with no antiphospholipid antibodies
SLE with positive antiphospholipid antibodies
Positive antiphospholipid antibodies

1
1
1
1

2
2
2
2

2
2
2
2

2
2
2
2

2
2
2
2

2
2
4
4

Cu-IUD copper intrauterine device, LNG-IUS levonorgestrel intrauterine system, IMP implanon/nexplanon, DMPA depot medroxyprogesterone acetate (IM and
SC), POP progestogen only pill, CHC combined hormonal contraception, VTE venous thromboembolism, I initiation of a method, C continuation of a method.
Reproduced under licence from FSRH. Copyright Faculty of Sexual and Reproductive Healthcare 2006e2016.

Table 9

Women with SLE are at increased risk of arterial and venous


thrombosis and this risk may be raised to an unacceptable level if
a woman with SLE, who has positive aPL, takes CHC. Because
POC does not increase the risk of arterial or venous thrombosis,
its use in SLE/APS is not contraindicated. The use of CHC should
be avoided in women with persisting positive aPL in the absence
of APS because of the potential risk of thrombosis (see Table 9).
Disease activity does not appear to be improved or made worse
by the use of hormonal contraception in women with SLE or
rheumatoid arthritis. Clinicians may wish to discuss that risk
with the individual womans specialist.

Immune system
HIV infection
The UK MEC guidance group concluded there was no robust
evidence that hormonal contraception or the copper IUD increases the risk of HIV acquisition or adversely affects the CD4
counts or viral loads of women with HIV infection. However,
clinicians may wish to avoid IUC insertion in women with a CD4
count <200 due to the possible risk of ascending genital tract
infection.
Some antiretrovirals affect the efficacy of contraceptives. Up
to date information on antiretroviral drug interactions can be
found on the Liverpool HIV Drug Interactions website http://
www.hiv-druginteractions.org.

Obesity
Obesity is a common problem in the UK. Obese women are more
likely to have an unplanned pregnancy than women with a
healthy BMI. Furthermore, obesity in pregnancy is associated
with an increased risk of miscarriage, stillbirth, pre eclampsia,
gestational diabetes, wound infections, congenital anomalies and
thromboembolism.
The risk of thromboembolism is the main concern in obese
women using contraception. Obesity is an independent risk factor for thromboembolism and, as already discussed, VTE rates
are overall higher in CHC users. There is evidence that CHC users
with higher BMIs have an increased risk of VTE compared to

Antiphospholipid antibodies
Antiphospholipid syndrome (APS) is an autoimmune condition
where individuals have raised antiphospholipid antibodies (aPL)
and a history of vascular (venous or arterial) thrombosis and/or
an adverse pregnancy outcome such as recurrent miscarriage,
intrauterine death, pre term delivery due to pre eclampsia or
placental insufficiency. APS can occur alone or in association
with conditions such as systemic lupus erythematosus (SLE) and
rheumatoid arthritis. apL without clinical manifestations are,
however, found in 5% of healthy individuals.

UK MEC categorization for women with obesity


Condition

Cu-IUD

LNG-IUS

IMP

DMPA

POP

CHC

BMI 30e34 kg/m2


BMI 35 kg/m2
History of bariatric surgery with BMI 30 kg/
m2
History of bariatric surgery with BMI 30e34
kg/m2
History of bariatric surgery with BMI 35 kg/
m2

1
1
1

1
1
1

1
1
1

1
1
1

1
1
1

2
3
1

Cu-IUD copper intrauterine device, LNG-IUS levonorgestrel intrauterine system, IMP implanon/nexplanon, DMPA depot medroxyprogesterone acetate (IM and
SC), POP progestogen only pill, CHC combined hormonal contraception, VTE venous thromboembolism, I initiation of a method, C continuation of a method.
Reproduced under licence from FSRH. Copyright Faculty of Sexual and Reproductive Healthcare 2006e2016.

Table 10

OBSTETRICS, GYNAECOLOGY AND REPRODUCTIVE MEDICINE 26:11

330

2016 Published by Elsevier Ltd.

REVIEW

medical conditions that preclude the use of all contraception. It


should be remembered that the UK MEC guidance relates only to
safety of contraceptive use and not to the effectiveness of
contraception or its use to treat medical conditions such as
endometriosis or PMS. LARC methods have the lowest pregnancy
failure rates and clinicians should encourage the use these
methods in all women.
A

CHC users with healthy BMIs and this increased risk is reflected
in the UK MEC guidance (Table 10).
Bariatric surgery includes restrictive procedures (laparoscopic
adjustable gastric banding and vertical sleeve gastrectomy),
malabsorptive procedures (biliopancreatic diversion with/
without duodenal switch), both restrictive and malabsorptive
procedures (roux-en-Y gastric bypass) and other procedures such
as gastric stimulation and intragastric balloon. The main concern
surrounding bariatric surgery and contraception is that of
malabsorption of oral methods and reduced effectiveness either
because of the surgery itself or post operative long term diarrhoea and vomiting. Women who have had bariatric surgery
should consider non oral methods if there are concerns over
absorption.

FURTHER READING
Faculty for Sexual and Reproductive Health. UK medical eligibility
criteria for contraceptive use. Clinical effectiveness unit 2016.
Available at:http://ukmec.pagelizard.com/2016.
Faculty of Sexual and Reproductive Health Statement. Venous
thromboembolism (VTE) and hormonal contraception. Clinical
effectiveness unit November 2014. Available at:http://www.fsrh.
org/documents/fsrhstatementvteandhormonalcontraceptionnovember/.
Faculty of Sexual and Reproductive Healthcare Clinical Guidance.
Contraceptive choices for women with cardiac disease. Clinical
effectiveness unit. 2014, https://www.fsrh.org/documents/ceuguidance-contraceptive-choices-for-women-with-cardiac/.
Faculty of Sexual and Reproductive Healthcare Clinical Guidance.
Drug interactions with hormonal contraception. Clinical effectiveness unit 2011. Available at:https://www.fsrh.org/documents/ceuguidance-drug-interactions-with-hormonal-contraception-jan/.
Faculty of Sexual and Reproductive Healthcare Clinical Guidance.
Emergency contraception. Clinical effectiveness unit 2011 (updated 2012), Available at:https://www.fsrh.org/standards-andguidance/documents/ceu-emergency-contraception-jan-2012/.

Emergency contraception
Emergency contraception (EC) can be offered to all women with
medical conditions. The most effective form of EC is the copper
IUD and should be offered first line. Alternatively, levonorgestrel
(LNG) or ulipristal acetate (UPA) can be used. Women taking
enzyme inducing medications should be given a double dose of
LNG rather than UPA. A Cochrane review found that women
who have more unprotected sex in same cycle after EC have a
three fold increased risk of pregnancy than women who do not
go on to have more unprotected sex. Therefore, when a woman
presents for EC she should be given effective contraception to
start immediately, ideally LARC. The FSRH document on Emergency Contraception gives detailed advice on the practicalities of
prescribing EC and on quick starting contraception.

Conclusion
Women with medical conditions may need to avoid or delay
pregnancy to prevent maternal and fetal morbidity. There are no

OBSTETRICS, GYNAECOLOGY AND REPRODUCTIVE MEDICINE 26:11

331

2016 Published by Elsevier Ltd.

CASE-BASED LEARNING

Problems in obstetric
anaesthesia

dilated cardiomyopathy (DCM) for which she is under the care of


a cardiologist and prior to her pregnancy was managed medically
with bisoprolol and perindopril which has now been changed to
bisoprolol and hydralazine. She also has an implantable cardioverter defibrillator (ICD) following two sustained episodes of
ventricular tachycardia (VT) around the time of her diagnosis.
She tells you that she does get short of breath but only if she is
exerting herself.
In the United Kingdom cardiovascular disease is the leading
cause of maternal mortality, and has remained so for more than
two decades. Over that period there has been an increase in
cardiovascular disease due to increasing maternal age and a
growing number of women who have had corrective surgery for
congenital cardiac abnormalities now reaching child bearing age,
the so called grown-up congenital heart (GUCH). Another
important, although rarer, group of women are those with cardiomyopathy, with an overall prevalence of just under 1 in 2000
deliveries. These comprise two distinct groups; those women
with a pre-existing cardiomyopathy, and those developing peripartum cardiomyopathy, typically either in late pregnancy or in
the early postnatal period.
The normal cardiovascular changes of pregnancy are well
documented with an increase in cardiac output of 40% arising
mostly from an increase in stroke volume and less significantly
an increase in heart rate. From early pregnancy, progesterone has
a vasodilating and fluid retaining effect helping to expand the
intravascular volume. Other important changes include a
decrease in both system vascular resistance and blood pressure.
Further demand is put on the cardiovascular system in the early
labour and particularly at the time of delivery with a further rise
in cardiac output of up to 50%.
A normal heart will cope with the cardiovascular changes
associated with pregnancy but a cardiomyopathic heart may not.
In patients with cardiomyopathy these changes can overwhelm
the functional reserve of the cardiovascular system which may
then result in decompensation and heart failure.
In this case we will look at the pathophysiology of dilated
cardiomyopathy and the measures that need to be taken in
pregnancy. We will also discuss the anaesthetic management
including consideration of patients with implantable cardioverter
defibrillators who need surgery.

Jonathan Lewis
Nav Bhandal

Abstract
Advances in medical management and technologies have increased
the prospects of women with severe chronic disease achieving pregnancy. Women with complex needs are becoming more commonplace
on labour suite and are posing an increasing challenge to healthcare
professionals. The management of dilated cardiomyopathy, postpartum headache and haemophilia are discussed using three theoretical cases. These cases illustrate the issues for both the anaesthetic
and obstetric teams and highlight the importance of a multidisciplinary
approach where good communication is essential in achieving the
safest possible outcome for such patients.

Keywords anaesthesia; cardiomyopathy; haemophilia; headache;


obstetric

Introduction
A common theme in all of the triennial reports, most recently
under the guise of Mothers and Babies: Reducing Risk through
Audits and Confidential Enquiries across the UK (MBRRACEUK), is that good quality, timely communication and multidisciplinary collaboration is the cornerstone of safe, effective practice. The reports from MBRRACE-UK, and previously from
CEMACH, have made a number of recommendations including
early consultation with a senior multidisciplinary team, use of
early warning scores in all obstetric units, adherence to obstetric
emergency protocols and early identification and targeted management of high-risk obstetric women including appropriate
escalation of care.
This review presents three hypothetical case scenarios to
illustrate some of the challenges encountered by the obstetric
anaesthetist as part of the multidisciplinary team caring for the
complex obstetric woman. An understanding of these issues
should inform the decision making process, in addition to promoting inter-specialty communication.

Pathophysiology of dilated cardiomyopathy


Dilated cardiomyopathy (DCM) is the commonest form of cardiomyopathy with a wide range of causes, of which 20e48% are
familial. Acquired causes include myocarditis, secondary to viral,
bacterial or protozoal infection, chronic alcohol or cocaine
misuse and autoimmune disease to name just a few. A significant
proportion of DCMs are idiopathic in origin, approximately 40%,
where no cause can be identified. Peripartum cardiomyopathy is
a subset of dilated cardiomyopathy affecting women in the late
third trimester and up to 5 months post delivery.
DCM is characterised by dilatation of the left ventricle with
associated systolic dysfunction in the absence of a hypertensive
or valvular cause. Affected individuals can go on to develop both
involvement of the right ventricle or diastolic dysfunction. These
structural changes to the heart increase the risk of developing
heart failure of either or both ventricles. In addition patients are

Case 1: cardiomyopathy and implantable cardioverter


debrillators
A 32-year-old woman in her first pregnancy presents to the
anaesthetic antenatal clinic. She has a 4 year history of idiopathic

Jonathan Lewis BMBS BSc (Hons) FRCA is a Fellow in Obstetric


Anaesthesia and Specialty Registrar at Nottingham University
Hospitals NHS Trust, Nottingham, UK. Conicts of interest: none
declared.
Nav Bhandal MBChB MRCP FRCA is a Consultant in Obstetric
Anaesthesia at Nottingham University Hospitals NHS Trust,
Nottingham, UK. Conicts of interest: none declared.

OBSTETRICS, GYNAECOLOGY AND REPRODUCTIVE MEDICINE 26:11

332

Crown Copyright 2016 Published by Elsevier Ltd. All rights reserved.

CASE-BASED LEARNING

also at risk of developing arrhythmias, particularly ventricular,


conduction disorders, syncope and sudden death.

to allow time for the woman to adjust to fluid shifts that result
from changing pre-load and after-load.
A well established epidural in labour can be cautiously topped-up for caesarean section in theatre with full monitoring. Use
of invasive arterial monitoring is advisable and central venous
access should be considered.
The patient presented in early labour at 34 weeks. She was
breathless on minimal exertion and had clinical evidence of
worsening heart failure with tachypnoea, bibasal crepitations and
ankle swelling.
Her arrival was communicated to the anaesthetic team who
sited an epidural early in labour. The cardiac physiology technicians were alerted to her presence as interrogation and deactivation of the implantable defibrillator would be needed in the event
of her needing to go to the operating theatre.

DCM and pregnancy


All patients with a pre-existing diagnosis of cardiomyopathy
should be offered pre-pregnancy counselling to discuss their
individualised risk based on history, examination and selected
investigations. It is important to establish the womans functional capacity. Examination may reveal signs of heart failure or
a cardiac dysrhythmia. A 12-lead ECG and echocardiogram
should be performed. Women who have a baseline functional
capacity of New York Heart Association (NYHA) class III or IV,
where they have functional limitation or shortness of breath after
less than normal activity, have an increased risk of adverse
outcomes during pregnancy as does having a left ventricular
ejection fraction less than 40%. A review of current medication is
also needed to change from those contraindicated in pregnancy
such as angiotensin converting enzyme inhibitors (ACE-I) and
anticoagulants.
For those women with no prior cardiac disease, diagnosis in
pregnancy can be very difficult. Many of the symptoms and signs
of heart failure, such as shortness of breath, ankle swelling and
orthopnoea are common in the course of a normal pregnancy. A
high index of suspicion is therefore needed not to miss either
patients with a newly developed cardiomyopathy or pre-existing
dysfunction unmasked by the cardiovascular changes of
pregnancy.

Implantable cardioverter debrillators


Although not as commonly used as pacemakers, insertion of ICD
is on the increase due to a broadening of the insertion criteria.
There are broadly two groups of patients for whom ICDs are
recommended.
 Patients with a previous serious ventricular arrhythmia
such as ventricular tachycardia (VT) or ventricular fibrillation (VF) not amenable to other treatment.
 Patients with a heritable cardiac disease with a high risk of
sudden death, e.g. long QT and Brugada syndromes, hypertrophic cardiomyopathy or arrhythmogenic right ventricular dysplasia.
In addition to the defibrillation function, ICDs may also have
the ability to pace, either for bradycardia or tachycardia, in much
the same way as a standard pacemaker. The commonest form of
device is implanted in the left pectoral region with trans-venous
electrodes but becoming increasingly common is a subcutaneous
version of the device with the electrode lying outside of the
thoracic cage.
When looking after a patient with an ICD it is important to
gain as much history about the device from the patient as
possible. Useful information includes the original indication for
insertion, frequency of counterchecks and device manufacturer
and model. The patient will often carry a card with much of this
information on it.
Carrying an ICD is safe during pregnancy and there are many
examples in the literature of discharging ICDs in pregnancy
without harm coming to the unborn fetus.
Transcutaneous electrical nerve stimulation devices (TENS)
should be avoided and magnetic resonance imaging (MRI) is
contraindicated.
If the woman needs to go to theatre the defibrillator and antitachycardia functions must be deactivated otherwise the device
can interpret interference from diathermy as VF and deliver a
shock. Precautionary trans-thoracic defibrillator pads should
ideally be attached to the patient prior to starting surgery.
Traditionally a magnet could be placed over these devices to
revert them to a basic pacemaker function and disable the defibrillator circuit however the effect of magnet placement in
modern devices varies, and formal device interrogation and
reprogramming by a technician is advised.

Management of this patient


This patient is known to have a history of DCM and should ideally
have received pre-pregnancy counselling. In the antenatal period
she should be managed by a multidisciplinary team comprising
obstetricians, cardiologists and anaesthetists. A thorough review
of her medical history should be undertaken to assess her risk
including a medication review. ACE-I are known to be linked to
congenital abnormalities and should be stopped. An alternative
vasodilator such as hydralazine should be considered.
Careful planning of her pregnancy and labour is necessary
involving cardiology, obstetric and anaesthetic input. Women
need regular reviews and, if prompted by a decline in functional
status or signs of worsening heart failure, investigations
including echocardiogram should be repeated. The third
trimester is usually the most challenging time for such women.
In the event that there is deterioration in the womans haemodynamic status that does not respond to medical management
a decision about the risks and benefits of early delivery will have
to be reached.
Women who are stable from a cardiac point of view can be
allowed to labour spontaneously, however if there are concerns,
a controlled induction of labour should be considered. Care
should be taken with prostaglandin E used for induction, as it is
possible to have systemic absorption and resultant cardiovascular effects. A shortened second stage of labour with assistance
with forceps or ventouse delivery minimises pushing and the risk
of a valsalva manoeuvre with its resulting circulatory effects.
An epidural sited in labour is recommended as it blunts the
physiological demands through reduction in somatic and sympathetic stimulation. The epidural should be established slowly

OBSTETRICS, GYNAECOLOGY AND REPRODUCTIVE MEDICINE 26:11

333

Crown Copyright 2016 Published by Elsevier Ltd. All rights reserved.

CASE-BASED LEARNING

Post operatively the device should be reactivated and tested


and the woman looked after in a setting with continuous cardiac
monitoring.
After the patient had an epidural sited it was carefully loaded
with anaesthetic to maintain stability and after a labour lasting
12 hours she had a forceps assisted vaginal delivery. She was
transferred to the high dependency unit for three nights before
eventually being discharged on day 6 post delivery with cardiology
follow-up.

Post dural puncture headache


More often than not an accidental breech of the dura mater
during epidural insertion is recognised, particularly if caused by
the Tuohy needle. A woman has approximately an 80% risk of
developing PDPH following a dural breech with a 16G Tuohy
needle. The headache is typically severe in nature with a strong
postural element, relieved by lying supine and exacerbated in the
sitting or standing position. It commonly presents within the first
72 hours following dural breech but the onset may be delayed by
some days or, rarely, by weeks. It is commonly associated with
tinnitus or hearing loss and neck stiffness. Treatment comprises
simple analgesics, ensuring the woman is well hydrated and
encouraging caffeine. Ultimately she may require an epidural
blood patch where around 20 ml of the patients own blood is
injected in the epidural space under strict aseptic conditions. The
optimal volume of injected blood is not known and commonly
the injection is continued until the woman experiences pain or
pressure in the back. A single epidural blood patch has a cure
rate between 50 and 95% and, if required, can be repeated
ensuring alternative diagnoses of headache have been
considered.

Case 2: postpartum headache


A 28-year-old woman with a history of migraine throughout
adulthood develops a severe headache. Five hours previously she
was delivered by emergency cesarean section for failure to progress due to malpresentation. She had an epidural sited in labour
for analgesia during which the dura was punctured with the
epidural catheter. The catheter was threaded into the intrathecal
space and later used in theatre for surgery. The catheter was
removed at the end of the operation.
Headache in the postoperative period is very common with a
reported incidence of 39% within the first week in one study.
Many minor headaches probably go unreported, being considered normal following delivery. The exact incidence is therefore
unknown. Causes of headache range from the simple, selflimiting headaches to the sinister and life threatening. It is
important to consider a wide differential when assessing such
women. In the MBRRACE-UK 2009e2012 triennial report two
patients died after their headaches were attributed to post dural
puncture headache despite persisting for some weeks. Their
deaths were attributed to subdural haematoma and a cerebral
venous sinus thrombosis.
Causes of headache can be divided into primary and secondary. Primary causes comprise migraine and tension headache, whereas the common secondary causes include post dural
puncture headache (PDPH), pre-eclampsia, subdural haematoma, cerebral venous thrombosis, meningitis, stroke and the
reversible cerebral vasoconstriction syndromes (RCVS).

Pre-eclampsia
Pre-eclamptic disease affects between 3 and 5% of pregnant
woman and the first 48 hours following delivery of the fetoplacental unit still carries significant risk. Headache related to preeclampsia may be accompanied by vomiting, visual disturbance and focal deficits. Patients may fit and are at risk of intracerebral haemorrhage from systolic hypertension. CT imaging
typically shows cerebral oedema. Management of pre-eclampsia
will be effective in reducing the symptoms.
Cerebral venous thrombosis
In both pregnancy and the immediate postnatal period, women are
in a hypercoagulable state. This combined with other risk factors
such as vomiting and dehydration can lead to an increased risk of
cerebral venous thrombosis. It can be difficult to diagnose as the
headache is often non-specific and may have a gradual onset.
There may be associated focal neurological signs or the woman
may fit. Diagnosis is established via a MR or CT venogram.

Diagnosing the cause of headache


The clinical context and history from the woman often provide
the crucial information to guide diagnosis. Enquiring about previous history of chronic headache is important as migraine is
common in woman of child bearing age and is associated with
further migrainous attacks, particularly in the early postnatal
period, as well as having an increased risk of pre-eclampsia,
eclampsia and stroke. Other associated symptoms should also be
sought.
This lady has risk factors for a number of different causes of
headache. She reports that her headache is worse when sitting and
almost completely relieved by lying flat on her bed. She complains
of hearing loss but no tinnitus. Examination does not reveal any
focal neurological signs or evidence of meningism. Her blood
pressure is elevated at 155/96. She receives intravenous paracetamol and oral morphine solution with some reduction in the
severity of her headache.
A consideration of some of the causes and characteristics of
postnatal headache will aid in narrowing down the differential
diagnosis and targeting investigations and treatment.

OBSTETRICS, GYNAECOLOGY AND REPRODUCTIVE MEDICINE 26:11

Meningitis
Although meningitis is an uncommon cause of postnatal headache the classic signs of meningism, photophobia in the presence
of pyrexia and sepsis should be looked for. Group B streptococcus in colonised women is thought to be the commonest
causative organism. Diagnosis is confirmed by microscopy and
culture of cerebrospinal fluid following a CT brain to exclude
other intra-cerebral pathology.
Reversible cerebral vasoconstriction syndrome
(RCVS)
Reversible cerebral vasoconstriction syndrome is a self limiting
syndrome of uncertain cause that is characterised by dysregulation of cerebral arterial tone. Pregnancy is a recognised precipitant with the symptoms occurring with a week of delivery. Other
known precipitating factors include SSRIs and ergot alkaloids.
Patients with migraine are believed to be more susceptible. The
headaches are described as thunderclap and may be associated

334

Crown Copyright 2016 Published by Elsevier Ltd. All rights reserved.

CASE-BASED LEARNING

conceptual, involving either testing for free fetal DNA in


maternal blood from 8 weeks, chorionic villus sampling from 11
weeks, or amniocentesis from 15 weeks gestation. Ultrasound
scanning for fetal gender is non-invasive alternative but not
normally performed until the detailed anatomy scan at 20 weeks
because of reduced reliability at earlier gestations.
Subsequent planning for delivery should be multidisciplinary
involving obstetricians, haematologists, neonatologist and
anaesthetists. Affected woman should be cared for and delivered
in a centre that is able to offer this service and the woman should
be seen regularly throughout pregnancy.
It is common in pregnancy to see a rise in factor VIII levels by
term up to three-fold from base levels. This is in contrast to factor
IX levels in haemophilia B which remain largely unchanged.
It is important to develop a birth plan for the woman but also
for the fetus, especially if male, with involvement of neonatal
services. Spontaneous delivery is safe and should be allowed
where possible. Consideration for elective caesarean section
should also be given. The woman should attend an anaesthetic
antenatal clinic to discuss and plan analgesia for labour.
Anaesthetic techniques for either operative delivery should be
outlined, in particular for the possibility of an emergency.
At 37 weeks the lady was admitted after spontaneously
rupturing her membranes at home. Her factor VIII levels taken in
pregnancy have shown a rise from 11 IU/dl pre-conception to 30
IU/dl when most recently tested at 34 weeks. She had a birth plan
for vaginal delivery with an open mind about analgesia. She had
bloods taken on arrival to labour suite including a full blood
count, coagulation screen, group and save and factor VIII levels,
which were found to be 37 IU/dl. The lady was found to have a
breech presentation which was subsequently confirmed by ultrasound scan. In accordance with haematology advice the patient
was given recombinant factor VIII.
If the factor VIII levels on arrival to labour suite are <50 IU/dl
then recombinant factor VIII should be administered, aiming for
levels of 100 IU/dl. Testing of levels after dosing should be
performed to confirm successful treatment. Epidurals and spinals
are safe to perform providing levels of factor VIII are >50 IU/dl.
They should be performed by an experienced anaesthetist as a
midline procedure to minimise the chance of bleeding.
The factor VIII levels increased to 108 IU/dl post treatment and
she was transferred to theatre for a caesarean section where she
was given a spinal anaesthetic with a 27G pencil point needle.
If an epidural catheter has been inserted factor VIII levels
should be checked prior to removal and again need to be >50 IU/
dl.
Other important aspects of anaesthetic care include the
maintenance of normothermia, avoidance of tachycardia and
hypertension and ensuring that the womans pressure points are
adequately padded and protected. Intramuscular injections are
safe provided levels of factor VIII are >50 IU/dl.
Avoidance of fetal scalp electrodes, fetal blood sampling and
rotational or mid-cavity forceps particularly when the fetus is
male is mandatory.
Following delivery factor VIII levels >50 IU/dl should be
maintained for 3 days following normal delivery and 5 days
following caesarean section. Thromboprophylaxis should be
considered on an individual basis. Non-steroidal anti-inflammatory drugs (NSAIDs) should be avoided.

with other symptoms including confusion, seizures and focal


neurological deficits. Spontaneous recovery can be expected after
2e3 months but it does have serious recognised complications,
namely, subarachnoid haemorrhage and stroke.
This lady had a headache typical of a postural puncture
headache. She was given simple analgesia and advice about
keeping well hydrated. However her headache did not resolve with
conservative management and she was successfully managed
with an epidural blood patch the following day. She was discharged home the same day and daily telephone follow-up
occurred until the headache was completely resolved.

Case 3: haemophilia
A 26-year-old nulliparous woman was referred to the anaesthetic
antenatal clinic by the obstetric team. She was known to have a
diagnosis of haemophilia A which was made following investigations into menorrhagia in adolescence. She had been taking oral tranexamic acid during her periods since diagnosis with
good effect. This was a planned pregnancy after pre-conceptual
counselling with a geneticist and an obstetrician and the lady
had undergone chorionic villus sampling (CVS) at 12 weeks. She
was expecting a female baby that had tested positive as a carrier
for haemophilia A.
The haemophilias are a group of genetic bleeding disorders
characterised by partial or complete deficiency in particular
clotting factors. It is subdivided into haemophilia A, B and C with
deficiencies in clotting factors VIII, IX and XI respectively. Both
haemophilia A and B display X-linked recessive inheritance
whereas haemophilia C is inherited as an autosomal recessive
disorder. 30% of patients, however, have no family history of
haemophilia and are affected as a result of new mutation. Haemophilia A is the commonest form with an incidence of
approximately 1 in 5000 live male births with the other two
forms rarer by an order of magnitude.
Haemophilia A and B, being X-linked recessive disorders,
affect males and rarely homozygous females. Normally females
are heterozygous for the mutation and are therefore only carriers,
and as a result only display a milder form of the disease. Their
offspring have a 50% chance of inheriting the affected X chromosome and therefore, if male, have a 50% chance of having the
disease, and, if female, have a 50% chance of being a carrier
themselves.
There is a wide spectrum of clinical severity which relates to
the level of active clotting factors. Levels are commonly
expressed in IU/dl with the normal range being 50e150 IU/dl of
normal. Patients with severe disease have levels <1 IU/dl,
moderate disease 1e5 IU/dl and mild disease 5e40 IU/dl.
Haemophilia A in pregnancy
Known or suspected carriers of haemophilia A should be offered
pre-conception counselling to discuss the risk of having a child
affected by haemophilia including alternatives to pregnancy.
Baseline factor VIII levels should be taken and the carrier status
of the woman tested if unknown. A discussion about different
methods of antenatal fetal sex determination, to identify males at
higher risk, should also take place. It is now possible to have preimplantation genetic diagnosis (PIGD) performed if also having
in vitro fertilisation (IVF). More commonly, testing is post

OBSTETRICS, GYNAECOLOGY AND REPRODUCTIVE MEDICINE 26:11

335

Crown Copyright 2016 Published by Elsevier Ltd. All rights reserved.

CASE-BASED LEARNING

The patient delivered a baby girl via caesarean section. Estimated blood loss was 950 ml. The baby was born in good condition with no signs of bruising or bleeding. The patient required
further doses of recombinant factor VIII over the next 5 days in
order to maintain levels >50 IU/dl and was then discharged home
with no further bleeding.

motherhood safer 2006e2008. The Eighth Report on Condential


Enquiries into Maternal Deaths in the United Kingdom. CEMACH,
2009.
Sabharwal A, Stocks GM. Postpartum headache: diagnosis and
management. Contin Educ Anaesth Crit Care Pain 2011; 11:
181e5.
Shah UJ, Narayanan M, Graham Smith J. Anaesthetic considerations
in patients with inherited disorders of coagulation. Contin Educ
Anaesth Crit Care Pain 2014; http://dx.doi.org/10.1093/bjaceaccp/
mku007.
Stergiopoulos K, Shiang E, Bench T. Pregnancy in patients with preexisting cardiomyopathies. JACC 2011; 58: 337e50.

Conclusion
Well planned multidisciplinary team working is the key to success when dealing with high risk, complicated obstetric patients.
They need input into their management from before conception
right through to the postnatal period. Planning for delivery is
important but maintaining flexibility in the face of a changing
clinical picture is also essential.
A

Practice points
FURTHER READING
Klein AM. Postpartum headache. IJOA 2010; 19: 422e30.
Knight M, Tuffnell D, Kenyon S, Shakespeare J, Gray R, Kurinczuk JJ
(Eds.) on behalf of MBRRACE-UK. Saving lives, improving mothers
care e Surveillance of maternal deaths in the UK 2011e13 and
lessons learned to inform maternity care from the UK and Ireland
Condential Enquiries into maternal deaths and morbidity 2009
e13. Oxford: National Perinatal Epidemiology Unit, University of
Oxford, 2015.
Lee CA, Chi C, Pavord SR, et al. The obstetric and gynaecological
management of women with inherited bleeding disorders e review
with guidelines produced by a taskforce of UK Haemophilia Centre
Doctors Organization. Haemophilia 2006; 12: 301e36.
Lewis G, ed. The Condential Enquiry into Maternal Deaths (CEMD)
saving mothers lives: reviewing maternal deaths to make

OBSTETRICS, GYNAECOLOGY AND REPRODUCTIVE MEDICINE 26:11

336

The haemodynamic changes in pregnancy may destabilise the


cardiomyopathic patient
The defibrillator and anti-tachycardia functions of an ICD need to
be deactivated prior to surgery. Using a magnet to achieve this is
not reliable with modern devices. It is essential to reactivate the
device immediately after surgery.
A headache that changes from one that is related to posture to
one that is constant in nature is a red flag and needs further
investigation
Haemophilia is not a contraindication to neuraxial anaesthesia
provided the levels of clotting factors are normal
Safe and effective management of the complex obstetric patient
requires careful planning and a multidisciplinary approach

Crown Copyright 2016 Published by Elsevier Ltd. All rights reserved.

CASE-BASED LEARNING

Induction of ovulation

the excess prolactin production is a pituitary microadenoma.


Treatment is with dopamine agonists and addressing the underlying cause. WHO type 3 anovulation (5%) (hypergonadotropic hypogonadism) is usually an indication of ovarian
failure.
Induction of ovulation is possible in the first two types of
anovulation. In the third type, ovulation induction is usually
unsuccessful due to follicular depletion and the only way to
achieve a pregnancy is through oocyte donation. The following
scenarios will explore the management of different causes of
anovulation more closely.

Alpha Gebeh
Mostafa Metwally

Abstract
Anovulatory infertility is one of the commonest causes of infertility and
can be caused by problems related to the ovary (normogonadotropic
and hypergonadotropic hypogonadism) or the pituitary and hypothalamus (hypogonadotropic hypogonadism). Consequently induction of
ovulation will depend on the cause of infertility. For those with normogonadotropic hypogonadism, ovulation can be induced using antioestrogens such as clomifene citrate and tamoxifen or aromatase
inhibitors such as letrozole. Second line treatments include metformin,
gonadotropins and laparoscopic ovarian drilling. Those with hypogonadotropic hypogonadism will require gonadotropins or GnRH analogues. The following review outlines the different approaches to
ovulation induction with a focus on commonly encountered clinical
scenarios.

Case 1
A 28-year-old woman with anovulatory infertility due to polycystic ovarian syndrome (PCOS) and a body mass index (BMI) of
40 kg/m2 has attended the infertility clinic to discuss options for
management of her infertility. Her partner has a normal semen
analysis and her hysterosalpingogram has demonstrated bilateral
patent tubes. She would like to discuss her options.
Discussion
Induction of ovulation for women with PCOS is the commonest
scenario for ovulation induction encountered in most infertility
clinics. Choice of treatment for this patient will depend on her
BMI. Weight loss is the first line of treatment in overweight and
obese PCOS women while Clomiphene citrate and Aromatase
inhibitors are recommended as the first line in lean PCOS ones.

Keywords anovulation; clomid; laparoscopic ovarian diathermy;


obesity; ovulation induction; PCOS

Introduction
Anovulatory infertility is one of the most commonly encountered
problems in the infertility clinic. Anovulation may be due to
problems affecting the ovary, pituitary or hypothalamus. Causes
of anovulation have been classified by the World Health Organisation into three categories based on gonadotropin profile e
follicle stimulating hormone (FSH), luteinizing hormone (LH)
and also oestradiol (E2). WHO type 1 anovulation (10%) is
characterized by a hypogonadotropic hypo-oestrogenic state and
includes hypothalamic amenorrhoea, hypogonadotropic hypogonadism and hypopituitarism. These can be caused by any
lesion affecting the pituitary or hypothalamus and affecting
gonadotropin production including idiopathic, weight-related
amenorrhoea, Sheehan syndrome, extreme stress and strenuous exercise, Kallmans syndrome, craniopharyngiomas etc.
WHO type 2 (normogonadotropic hypogonadism) is by far the
commonest cause of anovulation accounting for 85% of cases
and is most commonly caused by polycystic ovarian syndrome
(PCOS). Hyperprolactinaemic amenorrhoea is another, though
much less common, WHO Group II ovulation disorder. Clinically,
in addition to amenorrhoea and infertility, women with the
condition may have galactorrhoea. The most common source of

Weight loss: obesity is commonly associated with PCOS and can


have a detrimental effect on reproductive performance. The
central fat compartment is a metabolically active unit that produces a number of substances known as adipokines that can
adversely affect reproductive function. Obesity is associated with
resistance to ovulation induction using antioestrogens as first
line treatment, increases the gonadotropin doses required in
second line treatments and pose a higher technical risks if laparoscopy is needed down the line. Weight loss would both
improve fertility as well as decrease the risks that obesity may
pose to an ensuing pregnancy. Even a small decrease in the BMI
may be accompanied by a dramatic improvement in reproductive
performance. Strategies for weight loss include:
 Lifestyle interventions:
Multi-component interventions are the treatment of choice and
effective weight management programmes should include behavioural changes to increase the persons physical activity levels or
decrease inactivity. Behavioural changes include goal setting,
stimulus control, relapse prevention etc. Improved eating behaviour
and the quality of the persons diet are important as is reduction in
energy intake. However, unduly restrictive and nutritionally unbalanced diets are ineffective in the long term and can be harmful.
 Anti-obesity drugs:
Consideration should be given to pharmacological treatment
only after dietary, exercise and behavioural approaches have
been started and evaluated. Pharmacological agents are mainly
indicated when patients fail to lose significant weight despite
lifestyle changes and a low calorie diet or may be used to
maintain weight loss rather than for continued weight loss. There
are two main classes of drugs classified according to the site of

Alpha Gebeh DLM MRCOG PhD is a Subspecialty Trainee in


Reproductive Medicine and Surgery, The Jessop Wing and Royal
Hallamshire Hospital, Shefeld Teaching Hospitals, Shefeld, UK.
Conicts of interest: none declared.
Mostafa Metwally FRCOG MD is a Consultant in Reproductive
Medicine and Surgery, The Jessop Wing and Royal Hallamshire
Hospital, Shefeld Teaching Hospitals, Shefeld, UK. Conicts of
interest: none declared.

OBSTETRICS, GYNAECOLOGY AND REPRODUCTIVE MEDICINE 26:11

337

2016 Elsevier Ltd. All rights reserved.

CASE-BASED LEARNING

action; namely centrally and peripherally acting drugs. The only


available representative of the peripherally acting group is intestinal and pancreatic lipase inhibitor Orlistat, which acts by
decreasing fat absorption from the intestinal lumen by about
30%. Sibutramine, a centrally acting anti-obesity agent should
not be used as it has been withdrawn from the UK market after a
report from the European Medicines Agency that the cardiovascular risks associated with its use outweighed the benefits.
Furthermore, when choosing the appropriate weight-losing
drug, it is paramount to consider the safety of these drugs
should a woman conceive whilst receiving them. In this regard,
Orlistats pharmacokinetics places it in a favourable position due
to its low absorption and first-pass metabolism resulting in a
bioavailability of less than 1%.
Finally and regarding the efficacy of Orlistat in restoring
ovulation, current evidence suggests that Orlistat can lead to
similar ovulation rates to metformin when used over a threemonth period.

The main side effects of clomifene are related to its antioestrogen effects. Centrally this can lead to symptoms such as
hot flushes and uncommon but serious side effects include visual
symptoms such as blurred vision, diplopia, and photosensitivity.
Furthermore due to potential multifollicular development there is
an 8% chance of multiple pregnancies. Another potentially
serious but uncommon side effect is ovarian hyperstimulation
syndrome.
 Adjuvants:
Several adjuvants including growth hormone, ketoconazole
and dexamethasone have been used in addition to clomifene in
an attempt to improve ovulation and pregnancy rates but mostly
their use has not been supported by substantial evidence. However, metformin can be used alone or in combination with clomifene for ovulation induction in PCOS. Metformin is associated
with gastrointestinal side effects include nausea, vomiting and
abdominal upset. Dopamine agonists such as bromocriptine are
useful if there is associated hyper-prolactinaemia.

 Bariatric surgery:
As a final option, bariatric surgery can be used for those who
have failed to lose weight by other means. The guidelines of the
National Institute for Health and Clinical Excellence (NICE) states
that surgery is a treatment option for people with morbid obesity
(BMI equal to or greater than 40 kg/m2) or with a BMI equal to or
greater than 35 kg/m2 in the presence of significant co-morbid
conditions that could be improved by weight loss e.g. type 2
diabetes, hypertension. Surgical techniques include restrictive
procedures e.g. laparoscopic gastric banding, sleeve gastrectomy
or malabsorptive procedures e.g. Roux-en-Y gastric bypass. A
recent RCOG scientific opinion paper concludes that bariatric
surgery improves several important markers of fertility including
hyperinsulinaemia and ovulatory function. However, its potential
benefits need to be balanced against the risks of surgery.

Aromatase inhibitors: the aromatase inhibitors letrozole and


anastrozole have both been used to induce ovulation. They act
through the inhibition of ovarian aromatase enzyme with a
consequent decrease aromatisation of androgens into oestradiol.
This results in a decreased central negative feedback leading to
increased FSH production. Current evidence suggests that results
are similar to clomifene but with the potential advantages for
letrozole over clomifene include a shorter half-life that leads to a
preservation of the central feedback mechanisms and consequently monofollicular rather than multifollicular development.
The daily dose of letrozole in various studies has included 2.5
mg, 5 mg and 7.5 mg doses as well as a single 20 mg dose.
However current evidence suggests that the use of the 5 mg and
7.5 mg doses offer no significant improvement in the pregnancy
rates over the lower 2.5 mg dose.
Despite the effectiveness of letrozole in inducing ovulation, it
must be noted that it is not licensed for this particular use.
Furthermore the manufacturer states that there is a risk of fetal
anomalies as shown in animal studies although so far human
studies have been reassuring and have failed to show any
increased risk of fetal anomalies or adverse pregnancy outcomes.

Clomifene citrate: clomifene citrate, an oral selective oestrogen


receptor modulator is the first line pharmacological treatment
modality for women with anovulatory infertility (WHO type 2)
for over 40 years. Clomifene binds to oestrogen receptors in the
hypothalamus, releasing it from the negative feedback loop and
consequently leading to an increase in FSH that then leads to
follicular development.
Different doses of clomifene have been used and range from
50 mg to 150 mg per day but most commonly the starting dose is
50 mg per day with 50 mg increments per cycle until a maximum
dose of 150 mg per day given for 5 days. The start date of therapy
is usually within 5 days of the onset of menstruation or a
progestogen-induced withdrawal bleed but more commonly on
day 2 of the cycle. Those who do not respond to 150 mg per day
are considered to be clomifene resistant and consequently alternative treatments should be considered. Since the majority of
patients who will conceive will do so in the first three months,
other treatments should also be sought in those who do not
conceive after 6 months of therapy.
The rates of ovulation, pregnancy and live birth over a six
cycle of treatment are 49%, 30% and 23% respectively. This
relatively low conception rate may be due to the peripheral
antioestrogen effects of clomifene, particularly on the endometrium or due to the associated increased secretion of LH that can
lead to premature leutinisation of the developing follicle.

OBSTETRICS, GYNAECOLOGY AND REPRODUCTIVE MEDICINE 26:11

How to monitor response to treatment


Mid-luteal serum progesterone level is the most common method
of measuring success of ovulation induction. A sub-optimal level
indicates the need to increase the dose of the ovulation induction
agent, or failure of this mode of induction if the dose is already at
the maximum allowed. Commercial ovulation kits rely on serial
testing of urine for evidence of LH surge.
Serial ultrasonography is an efficient method of monitoring
follicular development and time ovulation in response to ovulation induction and in helping to reduce multiple pregnancy rates
especially in women with PCOS who are at risk of multifollicular
ovulation.

Case 2
The same patient had succeeded in dropping her BMI to 33 kg/
m2 and was commenced on a course of clomifene citrate. After 6
months she attended the clinic again as unfortunately clomifene

338

2016 Elsevier Ltd. All rights reserved.

CASE-BASED LEARNING

 Multiple pregnancies due to multifollicular development.


Obese women may require a higher dose of gonadotropins for
ovarian stimulation possibly due to poorer drug absorption as a
result of the increased subcutaneous fat compartment. However
this need for a higher dose of drugs should be dealt with
cautiously since these patients will commonly have PCOS making them highly sensitive to gonadotropin stimulation and prone
to ovarian hyperstimulation syndrome.

had been unsuccessful at inducing ovulation despite incrementally increasing the dose to 150 mg/day. She would like to
discuss further options.
Discussion
10e30% of women receiving clomifene will remain anovulatory
and are labelled as clomifene resistant. Options for second line
treatments include the use of insulin sensitising agents such as
metformin, the use of gonadotropins or laparoscopic ovarian
drilling.

Laparoscopic ovarian drilling: laparoscopic ovarian drilling


(LOD) can be used a second line treatment to induce ovulation in
women not responding to clomifene citrate. The procedure historically follows on from the earliest surgical procedure to induce
ovulation in women with PCOS involving wedge resection of the
ovary.
The exact mechanism by which LOD can induce ovulation in
not yet fully understood. Proposed mechanisms involve an
increased ovarian sensitivity to FSH, restoration of normal
ovarian-pituitary feedback mechanisms leading to a decrease in
baseline LH, a decrease in ovarian androgen production, and a
decrease in ovarian inhibin secretion leading to increased FSH
secretion.
Response to LOD has been found to be dependent on the
amount of energy delivered to the ovary where ovulation rates
increase with an increase in the dose of energy. Obviously this
needs to be balanced against the potential harm that can result
from the use of excess energy levels leading to ovarian damage.
Accordingly the ideal dosage needed is in the range of 600 J/
ovary.
Several factors have been found to influence the response to
LOD. The presence of elevated LH and low Anti-mullerian hormone (AMH) concentrations have been associated with favourable response to LOD while the presence of an increased BMI,
marked hyperandrogenism and long duration of infertility have
been found to predict resistance to treatment.
Compared to gonadotropin therapy, LOD can produce similar
ovulation and pregnancy rates but because it leads to monofollicular development, has the advantage of being associated
with a lower risk of multiple pregnancies. However the procedure is not devoid of problems. Apart from the obvious possible
complications of the laparoscopic procedure such as the risk of
visceral or vascular damage, LOD can be associated with the
occurrence of adhesions in a significant number of patients.
Furthermore there is also concern that LOD may damage the
ovarian reserve as evidenced by lower concentrations of AMH
and lower antral follicle counts following the procedure. However this evidence is not conclusive and could be interpreted as
merely a normalization of ovarian function rather than ovarian
damage. Nevertheless these potential risks suggest that minimising ovarian trauma during the procedure may be advantageous. Along these lines a few studies have suggested performing
unilateral rather than bilateral drilling after demonstrating both
techniques to be equally effective. However more studies are
needed before this becomes a widely accepted practice.

Insulin sensitising agents: the biguanide, metformin is the most


commonly used insulin sensitising agent in this respect and has
been used for some time particularly for patients with PCOS.
Initial reports suggested that metformin was highly successful
with a reported ovulation rate as high as 46%. Based on these
initial encouraging studies there was a wide interest in the use of
metformin even as an alternative first line therapy to clomifene.
Current evidence suggests similar chances of live birth with
clomifene or metformin and therefore suitable as second line
when clomifene cannot be used or unsuccessful. It is also biologically plausible that metformin co-treatment with clomifene
may increase the chances of live birth compared with the use of
either drug alone and therefore combined treatment is an alternative first or second line approach. However, the evidence is not
strong enough to make a firm recommendation that metformin
should be routinely used with clomifene to increase the chances
of a live birth. Metformin is given in doses of 1500e2000 mg in
divided daily doses and is associated with gastrointestinal side
effects.
Gonadotropins: gonadotropins may be used as a second line
treatment in patients with clomifene resistance or for those who
fail to conceive despite ovulating with clomifene. There are
several commercially available forms of FSH falling into two
main categories, urinary (u-FSH) and recombinant FSH (r-FSH)
and current evidence suggests that they are equally effective.
When administering gonadotropins for patients with PCOS it
is important to note that these patients are characterised by
certain factors that set them aside from other patients and need to
be taken into consideration when treating them with gonadotropins. Firstly they have an increased FSH threshold that is the
critical threshold level needed for to stimulate follicle recruitment. Secondly they have an increased sensitivity to exogenous
gonadotropins. These two factors can easily lead to ovarian hyperstimulation. To minimise the risk, a gentle stimulation
regimen such as a low dose step up regimen should be used.
Ultrasound monitoring of follicular growth is an integral part of
gonadotropin therapy to reduce the risks of multiple pregnancy
and ovarian hyperstimulation syndrome.
Complications of gonadotropin treatment
 Ovarian hyperstimulation syndrome: A potentially serious
condition characterised by shift of fluid from the intracellular to the extracellular compartment. The exact mechanism for the condition is unknown. The syndrome varies
in severity form mild to moderate to severe.
 Local allergic reactions
 Gastro-intestinal system disorders: nausea, abdominal
pain, pelvic pain.

OBSTETRICS, GYNAECOLOGY AND REPRODUCTIVE MEDICINE 26:11

Case 3
A 34-year-old patient has been diagnosed with secondary infertility. She has had one previous pregnancy that was delivered at

339

2016 Elsevier Ltd. All rights reserved.

CASE-BASED LEARNING

term by emergency caesarean section due to significant antepartum haemorrhage. After delivery she also suffered from
massive postpartum haemorrhage and needed 15 units of blood
transfusion for resuscitation. She was admitted to intensive care
but eventually fully recovered. She was unable to breast feed due
to lack of milk production and also suffered from secondary
amenorrhoea. Analysis of gonadotropins showed low concentrations of FSH and LH.

Other measures for ovulation induction in women with


WHO type 1 anovulation
Correction of eating disorders, increasing body weight to a
normal BMI (20e25 kg/m2) is required prior to ovulation induction. These lifestyle measures will help restoration of the
hypothalamoepituitary axis in women with hypothalamic amenorrhoea/anovulation.

Conclusions
Discussion
This represents a case of hypogonadotropic hypogonadism
(WHO type 1) due to massive postpartum haemorrhage resulting
in pituitary ischaemia and hypopituitarism (Sheehan syndrome).
Induction of ovulation in this case can be achieved with the use
of a combination of both FSH and LH since both are needed to
maintain normal follicular development. This combination can
be achieved either by the use of human menopausal gonadotropin (HMG) containing both products or by the use of r-FSH
combined with recombinant LH (r-LH).
The potential advantage of using r-FSH/r-LH is being able to
tailor the dosage to individual patient requirements as well as
more batch-to-batch consistency and a more predictable
response to dosage. Consequently r-LH can be combined with rFSH for ovarian stimulation usually at a dose of 150 IU r-FSH and
75 IU r-LH daily.

Anovulatory infertility is one of the commonest problems


encountered in an infertility setting. Most cases are normogonadotropic (WHO 2) and are most commonly due to PCOS. Induction of ovulation in women with PCOS is most commonly
with the use of antioestrogens such as clomifene as first line
treatment. Those who fail to ovulate with clomifene have the
option of surgical management with laparoscopic ovarian drilling
or further medical management with gonadotropins or metformin. For women with WHO type 1 anovulation (hypogonadotropic hypogonadism), treatment is most commonly through the
use of a combination of FSH and LH. Ovulation induction is not
usually possible with WHO type 3 anovulation (hypergonadotropic hypogonadism).
A

FURTHER READING
1 Balen A, Platteau P, Andersen AN, Devroey P, Helmgaard L,
Arce JC. Highly puried FSH is as efcacious as recombinant FSH
for ovulation induction in women with WHO Group II anovulatory
infertility: a randomized controlled non-inferiority trial. Hum Reprod
2007; 22: 1816e23.
2 Metwally M, Amer S, Li TC, Ledger WL. An RCT of metformin
versus orlistat for the management of obese anovulatory women.
Hum Reprod 2009; 24: 966e75.
3 Yasmin E, Davies M, Conway G, Balen A. Ovulation induction in
WHO type 1 anovulation: guidelines for practice. Produced on
behalf of the British Fertility Society Policy and Practice Committee.
Hum Fertil 2013; 16: 228e34.
4 National Institute of Clinical Excellence (NICE), CG156. Fertility
problems: assessment and treatment (London); 2013, https://www.
nice.org.uk/guidance/CG156.

The role of pulsatile gonadotropin releasing hormone


(GnRH) for ovulation induction
Gonadotropin releasing hormone is a decapeptide secreted from
the arcuate nucleus of the hypothalamus. The injection of GnRH
using a subcutaneous pump at a pulse frequency of 90e120
minutes can lead to induction of ovulation in about 50e80% of
cycles but only in women with an intact pituitary. Exogenous
GnRH therapy works best in hypogonadotropic hypogonadism
and restores a normal pulsatile GnRH rhythm allowing follicular
recruitment, selection, growth and development to proceed
similar to those of a normal menstrual cycle. When ovulation is
successful, cycle fecundability ranges from 10e30%. Pulsatile
GnRH have also been used in women with PCOS albeit not often.
However their effectiveness as second line therapy in PCOS remains uncertain as there is little evidence for their use compared
with other second lines of therapy such as gonadotropins. GnRH
infusion pumps are currently not available in many countries and
experience with their use remains limited.

Practice points

How to monitor response to gonadotropin treatment


Serial ultrasonographic folliculometry is the standard monitoring
method for Gonadotropin-based ovulation induction cycles. The
follicular size is correlated to the rising serum oestrogen levels as
a second parameter for monitoring. Ovulation is triggered when
the follicular size (approximately 18 mm mean follicular diameter) and serum oestrogen level indicate follicular maturity.

OBSTETRICS, GYNAECOLOGY AND REPRODUCTIVE MEDICINE 26:11

C
C

340

PCOS is the most commonly encountered cause of anovulation.


PCOS women are particularly prone to multifollicular development
and therefore a gradual step up regimen of stimulation with gonadotropins is usually recommended.
Weight loss in obese anovulatory women should be the main
focus of treatment and can lead to spontaneous ovulation.

2016 Elsevier Ltd. All rights reserved.

ETHICS/EDUCATION

Respectful care in labour

practitioner and patient as a means to achieve optimal outcomes.


The UK General Medical Council specifies that for good medical
practice we should respect patients right to reach decisions with
you about their treatment and care and provides a detailed
listing of requirements for practitioners on the elements that
must be demonstrated for them to establish and maintain
partnerships with patients. In parallel with these trends we have
also been challenged by midwives and by womens advocates as
to whether our professional attitudes and model of care are
actually fit for purpose. Most fundamentally the basic point is
made that childbirth is a normal bodily process and not a disease.
Doctors (or indeed any healthcare professional) do not have the
same privileged position of a doctorepatient relationship as very
often there is actually no patient, rather a woman experiencing a
normal event in her life. Midwives reflect this in their use of the
term client rather than patient. At the same time, as we know
the normal can very rapidly transform into an abnormal clinical
state and the client becomes a patient in an instant, requiring
timely and competent clinical intervention.
The history of maternity care includes many efforts to pre
select those at risk of complications during labour but, as we
know, our predictive abilities are strictly limited: most high risk
mothers do not actually suffer complications and as a result of
sheer numbers, most complications occur among those supposedly at low risk. This was reflected in the World Health Organizations slogan from the April 1998 World Health Day on Safe
Motherhood, Every pregnancy faces risk, and underpins the
international maternal health policy and programming emphasis
on assuring ready access to life saving interventions for all
mothers-to-be irrespective of risk status. For clinicians there is
therefore a need to provide care in labour that both responds to
the public health and statistical reality of ever present and unpredictable risk, while at the same time remembering that for
most people most of the time childbirth is a normal process, thus
requiring a service model that avoids inappropriate medicalisation and contains strong elements of humanistic and respectful care. These two requirements are not separate: if we fail to
offer services in a manner that women find acceptable they will
not use them and seek alternatives outside the formal health care
system, potentially putting themselves and their babies at risk.
This has been a major challenge in many low resource countries
where very high maternal mortality rates coincide with low utilisation of health centre or hospital maternity facilities.

William Stones

Abstract
Provision of respectful care in labour requires awareness of the special
circumstances of childbirth as a natural process. The need for woman
centred humanistic care runs alongside the duty to provide access to
clinical interventions as required, recognising the limitations of risk
assessment. Avoidance of over medicalisation and inappropriate interventions contributes to respectful care, as does leadership to develop
and sustain good working relationships among staff, so that women
experience care provided by staff who are functioning at their best.
Institutionalising respectful care provision alongside evidence based
clinical practice represents the goal of motherebaby friendly birthing
facilities as advocated by FIGO.

Keywords clinical standards; labour and delivery; maternity care;


reproductive rights

Why should it be necessary to present a review on respectful


care in labour to a readership of healthcare professionals?
Clearly none of us would intend to provide or indeed admit to
providing disrespectful care in the labour ward. Sadly, even in
well resourced settings women often report dissatisfaction with
their care and as medical directors and heads of department
around the world can testify, such experiences are often topics
for difficult meetings with mothers and their relatives following a
complaint. On such occasions the distress expressed can be just
as intense following an experience of disrespect, such as a
violation of privacy, of inadequate information provision or
inappropriate choice of words, as it can be following some major
medical mishap. This review aims to provide some reflections on
the topic of respectful care and will consider the contexts both of
well resourced health services but also those in low income
countries where provision of high quality care can be very
challenging. The review concentrates especially on the roles and
responsibilities of obstetricians in the promotion of respectful
care in labour.

The special circumstances of care in labour


In maternity care, clinicians have a different relationship with
women than they do in most medical encounters. The notion of
the privileged doctorepatient relationship is historically founded
on possession of technical knowledge and therapeutic skills
leading to a natural imbalance in authority, with language
reflecting that relationship as in doctors orders or indeed
prescription. The social, cultural and regulatory trend across
modern healthcare education and practice is to place increasing
emphasis on patient autonomy and a partnership between

Disrespectful care in labour: over medicalisation


The UK GMC states that practitioners must provide effective
treatments based on the best available evidence but in order to
achieve this, a combination of a supportive managerial and service context and optimal individual clinician behaviour is
needed. Engagement with advocates, users and clinicians is
needed to design and roll out service models that encourage
practitioners to use best evidence and avoid inappropriate interventions. In Eastern European countries the medical model is
still favoured and there have been recent instances of legal proceedings taken against midwives offering home birth and women
seeking this option, with the latter case reaching the European
Court of Human Rights. With regard to specific procedures that
are used excessively in medicalised childbirth, perhaps the best

William Stones MD FRCOG is Ann Gloag Chair of Global Health


Implementation at the University of St Andrews, UK and Departments
of Obstetrics and Gynaecology and Public Health, Malawi College of
Medicine, Malawi. Conicts of interest: none declared.

OBSTETRICS, GYNAECOLOGY AND REPRODUCTIVE MEDICINE 26:11

341

2016 Elsevier Ltd. All rights reserved.

ETHICS/EDUCATION

Towards the motherebaby friendly birthing facility

example is the persistence of routine episiotomy on the subcontinent and in China despite exhaustive systematic review
evidence against this approach.

In partnership with other international healthcare professional


associations and other stakeholders FIGO has set out the characteristics of a maternity unit that would meet the dual requirements of adherence to evidence based practice with
avoidance of inappropriate medical interventions, and assurance
of respect for womens rights and dignity during labour and delivery. Inspired by the long-established UNICEF Baby friendly
hospital model, the FIGO standards address aspects of care that
are all too frequently absent globally, for which there is no
justification for continued poor practice, but are easily remedied
through intentional leadership. A good example is allowing and
indeed encouraging companions to be present within delivery
rooms. They also address structural issues that may be more
challenging but are equally important for respectful care such as
avoidance of financial exploitation in maternity care. Applicable
equally to practice in wealthy and low resource countries there is
now a need to develop and document prototype motherebaby
friendly hospitals and birthing centres that can demonstrate how
respectful care can be institutionalised and made the new
normal for the benefit of all.
A

Respectful care in labour: fostering good practitioner


behaviour
In settings where disrespect and abuse of women in labour have
been identified, interventions to improve standards of behaviour
by providers have been shown to be effective, even in the
absence of major changes in staffing levels or resources at health
facilities. In Kenya, the Heshima (dignity) project used a multi
faceted approach working with policymakers to encourage
greater focus on disrespect and abuse, training providers on
respectful maternity care, and strengthening linkages between
the health facility and the surrounding community for accountability and governance. The findings following the intervention
were a reduction in the prevalence of mothers feelings of humiliation or disrespect from 20% to 13 % and there were statistically significant reductions in instances of physical abuse,
verbal abuse and violations of confidentiality.
In addition to the technical role, specialists have a responsibility for creating and supporting good practitioner
behaviour in their unit by role modelling and providing leadership to trainees and colleagues. This function is inseparable from
practitioners attitudes and behaviour towards colleagues: while
undermining and bullying can be found in any healthcare setting
or workplace in other sectors, these have been identified as
especially prevalent problems in obstetric units. In the UK,
recognition of this as a systemic problem led to work by the
RCOG and the Royal College of Midwives on the clinical learning
environment and an authoritative Joint Statement in 2015 with
supporting materials for training. It can be anticipated that
practitioners working effectively together across disciplines and
levels of seniority and demonstrating respect for each other will
not only improve the working environment for staff but translate
this into respectful care for women in labour.
Shared reflection and learning between practitioners caring
for women has a useful part to play in enhancing awareness of
womens reproductive rights and enabling practitioners to carry
this awareness into the clinical environment. FIGO has developed tools using case studies as a basis for reflection and discussion: most importantly, such activities undertaken in the
clinical environment will enable specific issues in the local
setting to be explored while engaging with universally applicable
principles of ethics and reproductive rights.

OBSTETRICS, GYNAECOLOGY AND REPRODUCTIVE MEDICINE 26:11

FURTHER READING
Abuya T, Ndwiga C, Ritter J, et al. The effect of a multi-component
intervention on disrespect and abuse during childbirth in Kenya.
BMC Pregnancy Childbirth 2015; 15: 224. http://dx.doi.org/10.
1186/s12884-015-0645-6.
 And Krejzova
 v.
European Court of Human Rights. Case of Dubska
2014. Strasbourg: The Czech Republic. Accessible at: http://
hudoc.echr.coe.int/eng?i001-148632.
FIGO Safe Motherhood and Newborn Health Committee, CM, White
Ribbon Alliance, International Pediatric Association, WHO. Motherbaby friendly birthing facilities. Int J Gynecol Obstet 2015; 128:
95e9.
FIGO. Womens rights, health and empowerment (web materials).
Accessible at http://www.glowm.com/womens_health_rights.
General Medical Council. The duties of a doctor. 2013. General
Medical Council. Accessible at: http://www.gmc-uk.org/guidance/
good_medical_practice/duties_of_a_doctor.asp.
Richmond D, Warwick C. The RCOG/RCM joint statement on undermining and bullying in the workplace. 2015. Royal College of Obstetricians & Gynaecologists. Accessible at: https://www.rcm.org.
uk/sites/default/les/Undermining%20Behaviours%20A4_7%20%
20-for%20website.pdf.

342

2016 Elsevier Ltd. All rights reserved.

SELF-ASSESSMENT

Self-assessment questions
Questions

1. A 29-year-old woman presents with a 6-month history of


increased thin, clear/white discharge that is most noticeable mid-cycle.
2. A 43-year-old presents to her general practitioner with
vaginal discharge. A swab is obtained and grows prominent clue cells, grade 3BV.
3. A 32-year old, para 2, whose last vaginal delivery was 2
years ago, has a HVS swab obtained as she complains of
white discharge. It comes back as positive for Streptococcus
agalactiae.
4. A 23-year old who is currently 18 weeks into her first
pregnancy has a swab obtained at her community midwife
as she has developed offensive discharge. The swab results
confirm the diagnosis of bacterial vaginosis.

SBA 1
A GP trainee currently on rotation in gynaecology asks for help
with understanding the UK MEC guidance in contraceptive
care. Which one of the following statements is accurate
regarding the MEC?
a) Reflects the effectiveness of contraception in women with
medical conditions.
b) Category 4 means there are no restrictions to use of the
contraception in women with that medial condition or
characteristic.
c) Relates to the use of the method for contraception and its
use in the management of medical disorders.
d) Category 3 means the theoretical or proven risks usually
outweigh the advantages of using the method.
e) Reflects the safety of contraception in women (except those
with chronic medical conditions).

EMQ 4
From the options below please select the most causative pathology or organism. Options may be used once, multiple time
or not at all.
a. Staphylococcus aureus
b. Vulvovaginal candidiasis
c. Candida glabrata
d. Trichomonas vaginalis
e. Bacterial vaginosis
f. Atrophic vaginitis
g. Gardnerella vaginalis
h. Group A streptococcus
1. A 42-year old woman attends the sexual health clinic
complaining of vaginal discharge. When a diagnosis is
reached she is informed that partner notification will be
required as part of management
2. Cause of recurrent candidiasis
3. A major cause of post-partum sepsis
4. Associated with an increased rate of preterm birth

SBA 2
A 34 year old woman who experienced a VTE following her
last pregnancy attends for contraceptive advice. Which of the
following statements is NOT true?
a) She should not use combined hormonal contraception.
b) She can be advised that the safest and most effective
method of contraception is IM/SC depot medroxyprogesterone acetate (DMPA).
c) She could have an IUD/IUS fitted if she is still taking
warfarin.
d) She can be advised that the advantages of using the progestogen only implant (nexplanon) generally outweigh the
theoretical or proven risks.
e) She should be counseled that 18 out of 100 women using
male condoms only for contraception will experience an
unintended pregnancy by 1 year.

SBA 5
You are the obstetrician serving as clinical director of a busy
hospital maternity unit. A patient has complained to the Chief
Executive about her care during labour and delivery and you
are asked to investigate and respond to the complaint. The
complaint letter refers to cleaning staff entering the labour
room without permission while Ms X was exposed, causing
her embarrassment. She also reports that while she was
pushing during the second stage, the midwife called a doctor
as she was concerned about the fetal heart rate. The doctor
thought it was normal and the two argued about whether
intervention was needed. She felt this behaviour was unprofessional. In the end she delivered normally without complications for her or the baby. Which one of the following options
would NOT form part of an appropriate investigation into this
complaint?
a) Review the case notes to identify any clinical issues
b) Confer with your midwifery management counterpart and
the manager responsible for housekeeping

EMQ 3
From the options below pick the single most appropriate
treatment for each scenario.
a. 2 g oral metronidazole
b. 500 mg PV clotrimazole
c. Nystatin 100,000 units four times per day for 3 weeks
d. No treatment required
e. Amphoteracin 1 mg IV daily for 3 days
f. Metronidazole 400 mg orally twice daily for 5 days
g. 150 mg fluconazole
h. Metronidazole 500 mg IV three times daily
i. Dermovate cream once daily for 4 weeks

Catherine Aiken PhD MRCOG is a Clinical Lecturer in Obstetrics


and Gynaecology at Addenbrookes Hospital, Cambridge, UK.
Conicts of interest: none declared.

OBSTETRICS, GYNAECOLOGY AND REPRODUCTIVE MEDICINE 26:11

343

2016 Elsevier Ltd. All rights reserved.

SELF-ASSESSMENT

d) Extreme weight loss


e) Extreme exercise

c) Obtain reports from the midwifery and medical staff


attending the patient
d) Review the protocols for non-clinical staff who need to
access clinical areas
e) Initiate disciplinary action against the staff involved

EMQ 9
From the options below pick the most appropriate treatment
for each scenario. Options can be used once, multiple times or
not at all.
a) Treat with dopamine agonists and address the underlying
cause
b) Start treatment with clomiphene citrate
c) Start treatment with aromatase inhibitors
d) Advise weight loss
e) Advise bariatric surgery
f) Switch to metformin treatment
g) Advise laparoscopic ovarian drilling
h) Start treatment with gonadotrophins
i) Start treatment with GnRH analogs
1. A 26-year old woman attends the infertility clinic with her
boyfriend of 2 years. They have been trying unsuccessfully
to conceive. On questioning, the patient is amenorrhoeic
and has noted a discharge from both breasts during the last
year.
2. A 30-year old woman attends the infertility clinic with her
partner. She reports oligomenorrhoea, hirsutism and acne.
Her body mass index is 38. They have been trying to
conceive for the last year.
3. A 36-year old woman with diagnosed PCOS has failed to
ovulate during several cycles of clomiphene citrate therapy. She has an extensive past history of bowel resection
for Crohns disease.

SBA 6
Having completed the initial investigation into the complaint
outlined in SBA 5, you respond to the complaint. Which one of
the following steps would NOT be an appropriate part of your
response?
a) Invite the patient and her partner to meet with you and the
midwifery manager.
b) Offer apology on behalf of the hospital.
c) Describe the steps taken to avoid a recurrence such as
feedback to the staff involved.
d) Note any additional concerns that the patient or her partner
might have about the service.
e) Explain that as you were not directly involved in the incident you are not best placed to respond to her concerns.
EMQ 7
From the options below pick the most appropriate treatment
for each scenario. Options can be used once, multiple times or
not at all.
a. 2 g oral metronidazole
b. 500 mg PV clotrimazole
c. Nystatin 100,000 units four times per day for 3 weeks
d. No treatment required
e. Repeat swabs in 10 days time
f. Metronidazole 400 mg orally twice daily for 5 days
g. 150 mg fluconazole
h. Daily application of oestrogen cream for 2 weeks
i. Dermovate cream once daily for 4 weeks
1. A 33-year old presents with increased white discharge with
associated vulval pruritus. This is her first presentation;
she has no other risk factors.
2. A 19-year old presents with lower abdominal pain and
profuse yellow, offensive discharge. On examination there
is generalised redness of the cervix and cervical excitation
is present. Multiple swabs are obtained including one for
wet microscopy; protozoa are seen. She is otherwise systemically well.
3. A 65-year woman who has recently become sexually active
with a new partner complains of vaginal discomfort during
and after intercourse, but no discharge.

SBA 10
A 28-year-old woman attends for an elective Caesarean section
at 39 weeks for a breech baby. She has hypertrophic cardiomyopathy and has an implantable cardioverter defibrillator
(ICD). Which of the following is true regarding her care
intraoperatively?
A) It is important to leave the device active during surgery in
case of dysrhythmias
B) A magnet placed near the device is reliable way of
disabling the defibrillator function
C) Patients require continuous cardiac monitoring postoperatively
D) The device is normally found subcutaneously in the right
pectoral region
E) Defibrillation ICDs cannot act as cardiac pacing devices

SBA 8
A 34-year old woman attends the infertility clinic with her
partner. They have been trying unsuccessfully to conceive for
over 2 years. Her periods have never been regular. Both her
gonadotrophin and oestradiol levels are very low on testing.
Which one of the following is not a possible cause of her
infertility?
a) Sheehans syndrome
b) Idiopathic hypogonadotrophic hypogonadism
c) Polycystic ovarian syndrome

OBSTETRICS, GYNAECOLOGY AND REPRODUCTIVE MEDICINE 26:11

SBA 11
A 38-year-old woman who is known to suffer from haemophilia A attends the antenatal clinic to plan delivery. Which
of one of the following statements is true about her
condition?
A) It is inherited via an X-linked dominant gene
B) Haemophilia A results in lower levels of coagulation factor
VIII
C) Haemophilia A is an absolute contraindication to epidural
analgesia

344

2016 Elsevier Ltd. All rights reserved.

SELF-ASSESSMENT

4. e
Bacterial vaginosis should be treated in pregnancy because
of the association with preterm birth

D) Intramuscular injections should not be given with factor


VII levels >50 iu/dl
E) Thromboprophylaxis is not required because of lower level
of coagulation factors

SBA 5 answer
e
In the initial investigation stages of the complaints process,
there should be no assumptions about where, if anywhere,
fault lies. Disciplinary action might be required at some stage,
but there is insufficient detail here to decide whether this is the
case.

Answers
SBA 1 answer
d
The UK MEC categorization reflects the safety of contraception in women with medical conditions. It does not reflect
the effectiveness of contraception in women with medical
conditions.

SBA 6 answer
e
As the clinical director, you have responsibility to represent
the unit whether or not you were directly involved. The initial
investigation should have involved establishing the facts as
clearly as possible. The other steps outlined are all part of the
routine process of responding to patient concerns and
complaints.

SBA 2 answer
b
Combined hormonal contraception is associated with an
increased risk of VTE and its use presents an unacceptable
health risk to women who have had a previous VTE. The
copper IUD is a UK MEC category 1 for women with previous
VTE and the IUS, progestogen only implant, DMPA and pop
are all category 2. The progestogen only implant would,
therefore, be the safest and most effective method of contraception for women with a history of VTE. Women should be
advised of the high typical failure rate of male condoms (18%)
and should be encouraged to use LARC. The IUD/IUS can be
fitted without interruption to anticoagulants so long as INR is
stable and in the target range.

EMQ 7 answers
1. b
This presentation is likely to be vaginal candidiasis, for
which pessary treatment with clotrimazole is appropriate
2. a
This patient has Trichomonas and should be treated with
metronidazole
3. h
This presentation is likely to reflect vaginal dryness from
lack of oestrogen

EMQ 3 answers
1. d
This discharge is most likely to be physiological
2. a
This is appropriate treatment for symptomatic bacterial
vaginosis
3. d
Strep agalactiae is a normal commensal species in the
vagina
4. f
This is an appropriate regimen for treatment of bacterial
vaginosis during pregnancy

SBA 8 answer
c
Polycystic ovarian syndrome is a WHO type 2 cause of
anovulation, in which normal gonadotrophin levels would be
expected. The other options listed are all causes of hypogonadotrophic hypogonadism.
EMQ 9 answers
1. a
The diagnosis here is hyperprolactinaemic amenorrhoea.
The patient will also need an MRI head looking for evidence of
a pituitary adenoma
2. d
This patient is very likely to have PCOS. The first line of
management should be weight loss, which is successful in
restoring ovulation in many patients.
3. h
This patient should move to a second-line ovulation
induction therapy. Addind metformin to clomiphene might be
a possibility, but not metformin alone. In a patient with
extensive previous surgery and likely adhesions, laparoscopy
should be avoided if possible.

EMQ 4 answers
1. d
Trichomonas is a sexually transmitted disease, where
partner notification is required
2. c
This contrasts to Candida albicans, which is more likely to
cause a single episode of vaginal candidiasis
3. h
Ascending genital tract infection with Group A
streptococcus postpartum is a major cause of maternal sepsis

OBSTETRICS, GYNAECOLOGY AND REPRODUCTIVE MEDICINE 26:11

345

2016 Elsevier Ltd. All rights reserved.

SELF-ASSESSMENT

SBA 10 answer
C

able to offer functions beyond defibrillation including antitachycardia pacing.

It is important to deactivate the defibrillator function on


ICDs prior to surgery as diathermy signals can be interpreted as ventricular fibrillation resulting in the device
delivering a shock. This is most reliably achieved through
reprogramming by a cardiac technician. After surgery the
device will need to be reactivated and then tested. Patients
should initially be cared for in an environment with
continuous cardiac monitoring. ICDs are most commonly
found in the LEFT pectoral region with newer devices
sometimes positioned over the ribcage. Modern ICDs are

SBA 11 answer
B
Haemophilia A is inherited via an X-linked recessive gene in
about 70% of cases and does lead to lower levels of coagulation
factor VIII. Provided that the levels of factor VIII are >50 iu/dl
both epidural insertion and intramuscular injection are safe. This
can be achieved by infusing recombinant factor VIII. After infusion of factor VIII, where levels have been corrected to normal,
patients are at risk of thrombotic events like other postpartum
women and therefore may require thromboprophylaxis.

OBSTETRICS, GYNAECOLOGY AND REPRODUCTIVE MEDICINE 26:11

346

2016 Elsevier Ltd. All rights reserved.

Vous aimerez peut-être aussi